69
7 3 i) ii) cδδ 0

Kako zapisati rje²enja? - Math · Kako zapisati rje²enja? IMO zadaci i bodovne sheme ... Nemojte zaboraviti trivijalne slu£ajeve (nulu, prazan skup, bazu indukcije) Pi²ite obja²njenja,

  • Upload
    others

  • View
    11

  • Download
    0

Embed Size (px)

Citation preview

Page 1: Kako zapisati rje²enja? - Math · Kako zapisati rje²enja? IMO zadaci i bodovne sheme ... Nemojte zaboraviti trivijalne slu£ajeve (nulu, prazan skup, bazu indukcije) Pi²ite obja²njenja,

IMO pripreme 2015. Matija Ba²i¢, 8. 6. 2015.

Kako zapisati rje²enja?

IMO zadaci i bodovne sheme

U ovom predavanju ¢emo pri£ati kako pristupiti zadacima na IMO.

Nekoliko op¢ih poruka:

• bolji natjecatelj se postaje isklju£ivo vjeºbanje, a kroz vjeºbu se moºe ve¢inastvari savladati

• bitno je znati osnovne rezultate i metode, ali vrlo speci£ni teoremi sukorisni samo u vrlo speci£nim situacijama

• najve¢a prepreka u rje²avanju teºih zadataka je psiholo²ka - ²to ranije ihpo£nete rje²avati, prije ¢ete se po£eti osje¢ati ugodno s tim nivom teºine

• ne postoji pravilo za uspjeh koje se moºe primijeniti na sve ljude, rezultatiovise o puno faktora, a pristup koji funkcionira je razli£it za razli£ite ljude- i to je super!

• neke ljude natjecanje motivira, dok druge pritisak blokira - osvjesti kako seti nosi² s pritiskom i prihvati to

• natjecanje traje dva dana, rezultat prvog dana (bio pozitivan ili negativan)ne treba stvarati pritisak na drugi dan ve¢ dati poticaj

• dajte sve od sebe, ali budite realni - sagledajte ²iru sliku i budite ponosnina svoj rezultat £ak i ako nije u skladu s po£etnim o£ekivanjem

• zapi²ite sve ²to uo£ite, ono ²to je tebi trivijalno drugima moºda nije

• bolje da zapi²ete jedan zadatak za 7 bodova, nego dva za 3, parcijalne bo-dove je te²ko dobiti, a lako izgubiti - urednim zapisivanjem rje²enja moºeteuo£iti gre²ke

• obavezno predajte sve papire na kojima ste pisali

• nemojte pisati s krive strane papira

• ako zapnete, prebaciti na kratko misli na ne²to drugo, pa krenite potpunoiz po£etka, na £istom papiru - uo£ite kad se vrtite u krug

• pazite da prelaskom na tvrdnju koju je dovoljno dokazati niste oti²li pre-daleko ili u krivom smjeru - ako ste zapeli provjerite originalnu formulacijuzadatku

• koristite oznake na logi£an na£in (nemojte imati istu oznaku za razli£itestvari, nemojte ozna£avati nabrajati slu£ajeve i), ii), c, δ, δ′, • )

• bolji natjecatelj se postaje prvenstveno vjeºbanje, a kroz vjeºbu se moºeve¢ina stvari savladati

Page 2: Kako zapisati rje²enja? - Math · Kako zapisati rje²enja? IMO zadaci i bodovne sheme ... Nemojte zaboraviti trivijalne slu£ajeve (nulu, prazan skup, bazu indukcije) Pi²ite obja²njenja,

Savjeti vezani uz bodovne sheme:

• Provjerite rje²enja jedndaºbi (funkcijskih, diofantskih, ...) - najgluplji na£inza izgubiti bodove je da vam nedostaje provjera

• Ako niste sigurni je li rezultat koji koristite poznat barem skicirajte dokaz(primjeri: triangulacije, simetrala kuta i simetrala nasuprotne stranice, x2+1 nema prost faktor oblika 4k + 3 itd.)

• Pazite na uvjete pod kojima skica u geometrijskom zadatku izgleda kako steju nacrtali, razlikujte slu£ajeve, nau£ite pravilno koristiti usmjerene kutove

• Zapi²ite ono ²to ste uo£ili na slici

• Rezultate koje dobijete analiti£ki ili kompleknim brojevima prevedite u ge-ometrijske tvrdnje - ako ne doete do kraja, takva rje²enja ne nose punobodova

• Nemojte zaboraviti trivijalne slu£ajeve (nulu, prazan skup, bazu indukcije)

• Pi²ite obja²njenja, jasno napi²ite denicije novih pojmova koje uvodite

• Ako znate dovr²iti zadatak pod dodatnim pretpostavkama, napi²ite taj dio

• Nemojte zaboraviti na indukciju

• Razmi²ljajte elementarno, tek ako zapnete po£nite razmi²ljati o teoremimakoji bi vam mogli pomo¢i

Zadaci za samostalan rad

1. U grupi od 25 biciklista ºelimo odrediti trojicu koja su najbrºa (tako daznamo tko je prvi, drugi i tre¢i). Na raspolaganju je staza na kojoj seistovremeno moºe utrkivati 5 biciklista. Ako je biciklist A u nekoj trci biobrºi od B, onda ¢e u svakoj trci biti tako. Ako je A bio brºi od B, a B brºiod C, onda ¢e i A biti brºi od C. Odredi najmanji broj utrka potrebnih dabi se odredila tri najbrºa biciklista.

2. U jednom vrhu kocke nalaze se dva pauka, a u suprotnom vrhu muha.Pauci i muha kre¢u se isklju£ivo po bridovima kocke jednakim konstantnimbrzinama. U svakom trenutku paucima je poznata pozicija muhe i muhi jepoznata pozicija pauka. Dokaºi da pauci mogu uhvatiti muhu. Smatra seda je muha uhva¢ena ako se nade u istoj to£ki kao i jedan od paukova.

3. Neka je n ≥ 3 prirodan broj. Ivan i Marija igraju sljede¢u igru: najprijeIvan ozna£i stranice pravilnog n-terokuta brojevima 1, 2, . . . , n na koji godna£in ºeli, koriste¢i svaki broj to£no jednom. Tada Marija dijeli n-terokutna trokute s n−3 dijagonale koje se ne sijeku u njegovoj unutra²njosti. Svete dijagonale ozna£ene su s brojem 1. U svaki od trokuta upisan je umnoºakbrojeva na njegovim stranicama. Neka je S zbroj tih n− 2 umnoºaka.

Odredi vrijednost od S ako Marija ºeli da je S najmanji mogu¢i, a Ivan ºelida je S najve¢i mogu¢i i oboje postupaju optimalno.

Page 3: Kako zapisati rje²enja? - Math · Kako zapisati rje²enja? IMO zadaci i bodovne sheme ... Nemojte zaboraviti trivijalne slu£ajeve (nulu, prazan skup, bazu indukcije) Pi²ite obja²njenja,

4. Na pravcu su ozna£eni brojevi 1,2,. . . ,2n u pravilnim razmacima. aba jena po£etku na broju 1, sko£ila je 2n − 1 puta i na svaki broj osim 1 jesko£ila to£no jednom. Ako je zbroj udaljenosti koje je presko£ila n(2n−1),na kojem je broju ºaba nakon svih skokova?

5. Neka je X skup od(

2k−4k−2

)+ 1 realnih brojeva, k > 2. Dokaºi da postoji

monotoni niz xiki=1 ⊂ X takav da je

|xi+1 − x1| > 2|xi − x1|,

za sve i = 2, . . . , k − 1.

6. U razredu postoji N u£enika razli£itih visina i teºina. Za grupu u£enikakaºemo da je usklaena ako za svaka dva u£enika u njoj vrijedi da je vi²iu£enik ujedno i teºi. S druge strane, za grupu u£enika kaºemo da je obrnutaako za svaka dva u£enika u njoj vrijedi da je vi²i u£enik ujedno i lak²i. Nekaje U veli£ina najve¢e usklaene grupe, a O najve¢e obrnute grupe. Dokaºiteda je U ·O > N .

7. Skup to£aka S u ravnini zovemo zdrug ako ne postoji kvadrat u danojravnini koji bi (u svojoj unutra²njosti ili rubu) sadrºavao sve to£ke od Sosim jedne. Dokaºite da svaki zdrug ima beskona£no mnogo to£aka.

8. Neka je n > 2 prirodni broj. Na plo£i dimenzija n × n postavljeno je kpijuna tako da svaki kvadrat dimenzija 2× 2 sadrºi to£no 2 pijuna. Odredisve mogu¢e vrijednosti za k.

9. Neka je S niz od 99 slova, i to 66 slova A i 33 slova B. Kaºemo da je Sdobar ako je za svaki inicijalni podniz od S (tj. niz koji se sastoji od prvihn slova u nizu S za 1 6 n 6 99) broj razli£itih rije£i koje moºemo dobitipermutiraju¢i slova u tom podnizu neparan. Odredi sve dobre nizove.

10. Neka je K skup svih bridova i dijagonala konveksnog 2015-terokuta u rav-nini. Kaºemo da je podskup A od K presijecaju¢i ako se svaki par duºinakoji pripada A sije£e. Odredi najve¢i mogu¢i broj elemenata u uniji dvapresijecaju¢a skupa.

11. Neka su m, n i r prirodni brojevi, pri £emu je n > 2 i 1 6 r 6 n−1. Neka jeplo£a dimenzija (mn+r)×(mn+r) prekrivena kvadratima dimenzija n×n(sa stranicama paralelnim stranicama plo£e). Svaki jedini£ni kvadrati¢ jeprekriveno barem jednom i neki su kvadrati¢i prekriveni vi²e puta. Odredinajmanji mogu¢i broj jedini£nih kvadrati¢a koji su prekriveni barem dvaputa.

Page 4: Kako zapisati rje²enja? - Math · Kako zapisati rje²enja? IMO zadaci i bodovne sheme ... Nemojte zaboraviti trivijalne slu£ajeve (nulu, prazan skup, bazu indukcije) Pi²ite obja²njenja,

IMO/MEMO pripreme 2015. Aleksandar Bulj

Algebarske manipulacije i nejednakosti

Uvod

Cilj ovog predavanja je pokazati na primjerima snagu algebarskih manipulacijau rje²avanju nekih vrlo te²kih natjecateljskih nejednakosti. U predavanju se nekoriste nikakve napredne metode niti alati matemati£ke analize. Jedini teoremi skojima bi £itatelj trebao biti upoznat su sljede¢i:

Teorem 1 (Nejednakosti sredina). Neka su a1, . . . , an pozitivni realni brojevi.Denirajmo za svaki realan broj k:

Sk(a1, . . . , an) :=k

√ak1 + ak2 · · ·+ akn

nza k 6= 0 i S0 = n

√a1 · a2 · · · an

Tada za m < n vrijedi Sm ≤ Sn. Uz jednakost kad je a1 = a2 = · · · = an

Teorem 2 (Cauchy-Schwarzova nejednakost). Neka su a1, . . . , an i b1 . . . bn realnibrojevi. Tada vrijedi nejednakost:

n∑i=1

aibi ≤

√√√√ n∑i=1

a2i

n∑i=1

b2i

Uz jednakost kad je a1 : a2 : . . . an = b1 : b2 : . . . bn.

Teorem 3 (Monotono preureenje vektora). Neka su a1 ≥ a2 ≥ · · · ≥ an i b1 ≥b2 · · · ≥ bn realni brojevi i neka je σ : 1, 2, . . . , n → 1, 2, . . . , n permutacija.Tada vrijedi:

n∑i=1

aibi ≥n∑i=1

aibσ(i) ≥n∑i=1

aibn+1−i

Teorem 4 (ebi²evljeva nejednakost). Neka su a1 ≥ a2 ≥ · · · ≥ an i b1 ≥b2 · · · ≥ bn realni brojevi. Tada vrijedi:

n

n∑i=1

aibi ≥n∑i=1

ai

n∑i=1

bi ≥ n

n∑i=1

aibn+1−i

Dokaze ovih teorema lako je na¢i na internetu tako da ovdje nisu navedeni. Radikra¢eg i preglednijeg zapisa, u daljnjem ¢e tekstu znakovi

∑i∏

bez indeksaozna£avati cikli£ku sumaciju odnosno cikli£ni produkt.

Page 5: Kako zapisati rje²enja? - Math · Kako zapisati rje²enja? IMO zadaci i bodovne sheme ... Nemojte zaboraviti trivijalne slu£ajeve (nulu, prazan skup, bazu indukcije) Pi²ite obja²njenja,

Rije²eni zadaci:

Zadaci su poslagani po teºini. Meutim, teºina zadatka je subjektivna ocjenapa probajte o svakome zadatku barem 15min sami razmisliti da znate u £emunjegova teºina. Neka vas ne razo£ara ukoliko ne uspijete rije²iti sve zadatke jersu neki vrlo te²ki.

1. Neka su a, b, c realni brojevi, takvi da je ab + ac + bc = 0. Dokaºite davrijedi:

a2(1− b) + b2(1− c) + c2(1− a) ≥ abc(a+ b+ c− 3)

Rje²enje. U ovom zadatku vidimo da homogenizacija ne¢e pomo¢i papogledajmo kako bismo jo² mogli iskoristiti uvjet. Uo£itmo da se izrazabc(a+ b+ c) pojavljuje u raspisu (ab+ac+ bc)2 pa poku²ajmo to ikoristiti:abc(a+ b+ c) = −1

2

∑(ab)2. Uvr²tavaju¢i to u nejednakost, ostaje nam za

dokazati: ∑(2a2 + a2b2 − 2a2b+ 2abc) ≥ 0

Ovaj izraz sad ve¢ jako podsje¢a na izraz koji dobijemo u kvadratu trinomapa poku²ajmo namjestiti izraz na to oduzimanjem 2(ab+ ac+ bc) od lijevestrane: ∑

(a2b2 + a2 + c2 − 2a2b− 2ac) =∑

(ab+ c− a)2 ≥ 0

£ime je dokaz zavr²en. Jednakost se o£ito postiºe za a = b = c.

2. Neka su a, b, c, d realni brojevi, takvi da je (1+a2)(1+b2)(1+c2)(1+d2) = 16.Dokaºite da vrijedi:

−3 ≤ ab+ ac+ ad+ bc+ bd− abcd ≤ 5

Rje²enje. U ovom zadatku imamo vrlo nestandardan uvjet, koji je neho-mogen i s kojim, na prvi pogled, ne moºemo ni²ta. Meutim, £esto takviuvjeti sugeriraju rje²enje. Prisjetimo se vrlo korisnog identiteta u kojem sepojavljuje umnoºak sume dva kvadrata:

(x2 + y2)(z2 + w2) = (xz + yw)2 + (xw − yz)2

Vidimo da je ovime zadatak skoro gotov. Zapi²imo uvjet u obliku:∏(1 + a2) =

[(a+ b)2 + (1− ab)2

] [(c+ d)2 + (cd− 1)2

]Koriste¢i Cauchy-Schwarzovu nejednakost znamo da vrijedi:

16 =[(a+ b)2 + (1− ab)2

] [(c+ d)2 + (cd− 1)2

]≥ [(a+ b)(c+ d) + (1− ab)(cd− 1)]2

Odakle raspisivanjem desne zagrade slijedi tvrdnja.

Page 6: Kako zapisati rje²enja? - Math · Kako zapisati rje²enja? IMO zadaci i bodovne sheme ... Nemojte zaboraviti trivijalne slu£ajeve (nulu, prazan skup, bazu indukcije) Pi²ite obja²njenja,

3. Neka su a, b, c, d ≥ 0realni brojevi takvi da je a2 +b2 +c2 +d2 = 4. Dokaºiteda vrijedi:

1

5− a+

1

5− b+

1

5− c+

1

5− d≤ 1

Rje²enje. Jedna, moºda nestandardna, ideja je zapisati nejednakost uobliku:

∑ 1− a5− a

≥ 0

Sada primjetimo da znamo da je∑

(1− a2) = 0 pa ¢emo probati to dobitiu brojniku. To radimo zbog toga ²to ebi²evljeva nejednakost kaºe da jesuma nekih umnoºaka (pomnoºena s konstantom) ve¢a ili jednaka umno²kusuma pa ¢emo na taj na£in probati "izolirati" sumu brojnika.

∑ 1− a5− a

=∑ 1− a2

(5− a)(1 + a)

Sad jo² samo treba provjeriti da su funkcije: 1−x i 1(5−x)(1+x)

obje padaju¢ena intervalu [0, 2], ²to je trivijalno. Kona£no, iz ebi²evljeve nejednakostivrijedi:

∑ 1− a2

(5− a)(1 + a)≥ 1

3

∑(1− a2)

∑ 1

(5− a)(1 + a)= 0

*Napomena: Ova nejednakost moºe se pokazati i kori²tenjem ograde:

1

5− x≤ x2

32+

7

32

koja vrijedi za x ∈ [0, 2], meutim to nije dio ovog predavanja.

4. Neka su a1, . . . an, b1, . . . bn ∈ R takvi da je

n∑i=1

a2i =

n∑i=1

b2i = 1 i

n∑i=1

aibi = 0

Dokaºite da vrijedi:

(n∑i=1

ai

)2

+

(n∑i=1

bi

)2

≤ n

Rje²enje. Kad bismo primjenili Cauchy-Schwarzovu nejednakost odmah nana lijevu stranu, ograda bi bila preslaba. Meutim, te²ko je i£im drugimocijeniti lijevu stranu pa ostajemo kod te ideje, samo je malo izmijenimo.Uvedimo oznake: A =

∑ni=1 ai i B =

∑ni=1 bi. Ako su i A i B, jednaki 0,

nejednakost vrijedi pa pretpostavimo da je barem jedan razli£it od 0. Tadaiz Cauchy-Schwarzove nejednakosti slijedi:

Page 7: Kako zapisati rje²enja? - Math · Kako zapisati rje²enja? IMO zadaci i bodovne sheme ... Nemojte zaboraviti trivijalne slu£ajeve (nulu, prazan skup, bazu indukcije) Pi²ite obja²njenja,

(A2 +B2)2 =

(n∑i=1

(aiA+ biB)

)2

≤ n

n∑i=1

(aiA+ biB)2 = n(A2 +B2)

Dijeljenjem obje strane s (A2 +B2) dobivamo traºenu nejednakost. Primje-timo da se jednakost postiºe npr. za a1 = 1, a2 = · · · = an = 0, b1 = 0,b2 = · · · = bn = 1√

n−1.

*Napomena 1: Nejednakost se moºe dokazati i namje²tanjem pogodnogskalarnog produkta jer vidimo da uvjet moºemo zapisati vektorski na na£inda uzmemo a = (a1, . . . , an), b = (b1, . . . , bn) za koje vrijedi: 〈a, a〉 =〈b, b〉 = 1 i 〈a, b〉 = 0

*Napomena 2: Poku²ajte na sli£an na£in dokazati nejednakost Minkowskog

5. Neka su a, b, c > 0 konstante i x, y, z > 0 realni brojevi takvi da je ax +by + cz = xyz. Dokaºite da postoji jedinstven realan broj d takav da je

2

d=

1

a+ d+

1

b+ d+

1

c+ d

i dokaºite da je tada minimum izraza x+y+z jednak√d(d+ a)(d+ b)(d+ c).

Rje²enje: esto je dobro homogenizirati nejednakost pa napravimo to. Tvr-dimo da je minimum izraza:

(x+ y + z)2(ax+ by + cz)

xyz(1)

gdje su sad x, y, z > 0 proizvoljni realni brojevi jednak traºenom minimumu(Za²to?). Kad smo se rije²ili "£udnog" uvjeta, imamo puno alata za rje²a-vanje ovakvih problema. to se prvog dijela zadatka ti£e, tvrdnja slijedi iztoga ²to je funkcija f(x) = x

a+xrastu¢a na [0,+∞) za a > 0. to se drugog

dijela ti£e, ideja je primjeniti AG na (1), ali malo "modiciran" da bismoo£uvali uvjet jednakosti. Neka su m,n, p,m1, n1, p1 pozitivni brojevi, takvida je m+ n+ p = am1 + bn1 + cp1 = 1.Iz AG-a vrijedi:

x+ y + z = m · xm

+ n · yn

+ p · zp≥ xmynzp

mmnnpp(2)

i

ax+ by + cz = m1 ·ax

m1

+ n1 ·by

n1

+ p1 ·cz

p1

≥ xam1ybn1zcp1

mam11 nbn1

1 pcp11

(3)

Iz (2) i (3) slijedi:

(x+ y + z)2(ax+ by + cz) ≥ x2m+am1y2n+bn1z2p+cp1

m2mn2np2pmam11 nbn1

1 pcp11

Gdje jednakost vrijedi za xm

= yn

= zpi xm1

= yn1

= zp1

Page 8: Kako zapisati rje²enja? - Math · Kako zapisati rje²enja? IMO zadaci i bodovne sheme ... Nemojte zaboraviti trivijalne slu£ajeve (nulu, prazan skup, bazu indukcije) Pi²ite obja²njenja,

Sada, ºelimo namjestiti koecijente tako da vrijedi:

2m+ am1 = 2n+ bn1 = 2p+ cp1 = 1

Stavimo k = 2mm1

= 2nn1

= 2pp1. Tada vidimo da vrijedi:

2am+ 2bn+ 2cp = k

i

2m(ak

+ 1)

= 2n

(b

k+ 1

)= 2p

( ck

+ 1)

= 1

Iz ova 2 uvjeta slijedi kona£no:

ak

a+ k+

bk

b+ k+

ck

c+ k= k ⇔ 1

a+ k+

1

b+ k+

1

c+ k=

2

k

Kako je rje²enje ove jednadºbe jedinstveno, vrijedi da je k = d i jednostav-nim ra£unom dobijemo

m2mn2np2pmam11 nbn1

1 pcp11 =1

d(d+ a)(d+ b)(d+ c)

Odakle slijedi tvrdnja zadatka.

6. Neka su a, b, c, d, e ≥ 0 realni brojevi, takvi da je a + b + c + d + e = 5.Dokaºite da vrijedi:

abc+ bcd+ cde+ dea+ eab ≤ 5

Rje²enje. Kod ovog zadatka gotovo svi standardni pristupi, u kojima pri-mjenjujemo nejednakost na sve £lanove, ne prolaze pa je potrebno malovi²e "analiti£ki" pristupiti zadatku i pametno transformirati izraz. Kakoto napraviti? Vrlo £esto je dobro primjenjivati nejednakosti na sumu ne-kih brojeva umjesto na svaki £lan posebno jer se sume "pona²aju" bolje odbrojeva ²to se jednakosti ti£e (ne moraju svi £lanovi suma biti jednaki dabi dvije sume bile jednake).

To nas vodi na ideju da poku²amo barem djelomi£no faktorizirati nejed-nakost da bismo onda primjenili AG nejednakost, koja nam kaºe da jeumnoºak ne£ega manji do sume. Bez smanjenja op¢enitosti, smijemo pret-postaviti da je e najmanji broj. Sada iz AG nejednakosti vrijedi:

abc+ bcd+ cde+ dea+ eab = e(a+ c)(b+ d) + bc(a+ d− e)

≤ 1

4e · (a+ b+ c+ d)2 +

1

27(a+ b+ c+ d− e)3 =

1

4e(5− e)2 +

1

27(5− 2e)3

Dakle, ostaje za dokazati da je: 14e(5− e)2 + 1

27(5− 2e)3 ≤ 5.

Meutim, to je ekvivalentno s: (e − 1)2(e + 8) ≥ 0, pa je nejednakostdokazana.

Page 9: Kako zapisati rje²enja? - Math · Kako zapisati rje²enja? IMO zadaci i bodovne sheme ... Nemojte zaboraviti trivijalne slu£ajeve (nulu, prazan skup, bazu indukcije) Pi²ite obja²njenja,

7. Odredi najmanji realan brojM takav da za sve realne brojeve a, b, c vrijedi:

|ab(a2 − b2) + bc(b2 − c2) + ca(c2 − a2)| ≤M(a2 + b2 + c2)2

Rje²enje. Ovdje je vrlo o£ito da se lijeva strana nejednakosti treba faktori-zirati pa najprije napravimo to. Dakle, nejednakost je ekvivalentna s:

|(a+ b+ c)(a− b)(b− c)(a− c)| ≤M(a2 + b2 + c2)2

Sada primjetimo da na lijevoj strani imamo umnoºak nekih £lanova, a nadesnoj zbroj. To nam sugerira da ¢emo koristiti AG nejednakost. Za to bibilo dobro da su pribrojnici na desnoj strani isti kao i £lanovi umno²ka nalijevoj strani. Zbog toga ¢emo transformirati nejednakost u oblik:

|(a+b+c)(a−b)(b−c)(a−c)| ≤ M

9((a−b)2+(b−c)2+(c−a)2+(a+b+c)2)2

Radi preglednosti, sad moºemo staviti da je x = a− b, y = b− c, z = c− ai s = a+ b+ c. Primjetimo da je x+ y + z = 0, a nejednakost postaje:

|xyzs| ≤ M

9(x2 + y2 + z2 + s2)2

Ovo je ve¢ puno bliºe obliku AG nejednakosti na 4 £lana. Meutim, mo-ramo paziti na uvjet x+y+z = 0 zbog kojeg ne moºemo direktno primjenitiAG. Meutim, kako je poe£tna nejednakost simetri£na (zbog apsolutne vri-jednosti), bez smanjenja op¢enitosti moºemo pretpostaviti a ≥ b ≥ c, tj.x, y ≥ 0. Rije²imo se sad i zadnjeg uvjeta i stavimo da je z = −x−y Dakle,nejednakost postaje:

|xy(x+ y)s| ≤ M

9(x2 + y2 + (x+ y)2 + s2)2

Kona£no, pretpostavimo da ¢e se jednakost postizati kad je x = y pa pri-mjenimo AG nejednakost najprije za 4 £lana pa na 2 £lana na na£in:

((x2 + y2) + 2 ·

(x+ y√

2

)2

+ s2

)2

≥ 16·√x2 + y2

(x+ y√

2

)2

|s| ≥ 16√

2|xy(x+y)s|

Provjerimo moºe li se posti¢i jednakost. Iz slu£ajeva jednakosti u AG-udobivamo da mora vrijediti x = y = s√

2, tj. a − b = b − c = a+b+c√

2odakle

lako slijedi da se jednakost postiºe npr za: (a, b, c) =(

3√

2+22

, 1, 2−3√

22

).

Dakle, ograda je stroga i vrijedi M = 916√

2.

Page 10: Kako zapisati rje²enja? - Math · Kako zapisati rje²enja? IMO zadaci i bodovne sheme ... Nemojte zaboraviti trivijalne slu£ajeve (nulu, prazan skup, bazu indukcije) Pi²ite obja²njenja,

8. Neka su a, b, c realni brojevi. Dokaºi da vrijedi:

(a2 + b2 + c2)2 ≥ 3(a3b+ b3c+ c3a)

Rije²enje. Ovaj zadatak, iako izgleda jednostavno, izuzetno je teºak, alipokazat ¢emo kako do rje²enja do¢i metodi£no. Za neke korake je potrebnopuno iskustva u algebarskim manipulacijama. itatelju je vjerojatno poz-nata metoda zapisivanja rje²enja pomo¢u sume kvadrata i to je vjerojatnone²to £ime se u nekom trenutku proba rije²iti ovaj zadatak. Meutim ovdjeto ne pomaºe, ali to ¢e nam dati naznaku kako krenuti. Transformirajmonajprije nejednakost u oblik:

∑a4 −

∑a2b2 ≥ 3

(∑a3b−

∑a2b2

)Vidimo da se sada lijeva strana moºe zapisati u obliku sume kvadrata, alidesna nije postala ni²ta "bolja". Ideja zapisa u obliku sume kvadrata namdaje ideju da desnu stranu jo² transforiramo tako da oduzmemo i dodamosumu koja je jo² manja od

∑a2b2, a to je

∑a2bc da bismo dobili jo² jednu

sumu kvadrata na lijevoj strani. Tada nejednakost postaje:

∑a4 −

∑a2b2 + 3

(∑a2b2 −

∑a2bc

)≥ 3

(∑a3b−

∑a2bc

)Zapi²imo nejednakost na na£in:

1

2

∑(a2 − b2)2 + 3

(∑a2b2 −

∑a2bc

)≥ 3

∑ab(a2 − c2)

Jo² nam ostaje srediti drugi. O£ito je da je on pozitivan, ali ako ga raspi²emokao sumu kvadrata na analogan na£in kao ²to smo prvi £lan, ne dobijemoni²ta korisno. U ovom slu£aju, morat ¢emo biti "ma²tovitiji" i poku²ati sasli£nim identitetom:

∑x2 −

∑xy =

1

6

∑(x+ y − 2z)2

Nejednakost sada moºemo zapisati u obliku:

1

2

∑(a2 − c2)2 +

1

2

∑(ac+ bc− 2ab)2 ≥ 3

∑ab(a2 − c2)

Ovo nas, kona£no, podsje¢a na kvadrat binoma, ali "nedostaju" neki £lanovina desnoj strani. Ipak, ako primjetimo da vrijedi:

0 = (ab+ ac+ bc)∑

(a2 − c2) =∑

(ab+ ac+ bc)(a2 − c2)

i oduzmemo to od desne strane, nejednakost se svodi to£no na oblik zasumu kvadrata binoma na na£in:

Page 11: Kako zapisati rje²enja? - Math · Kako zapisati rje²enja? IMO zadaci i bodovne sheme ... Nemojte zaboraviti trivijalne slu£ajeve (nulu, prazan skup, bazu indukcije) Pi²ite obja²njenja,

∑(1

2(a2 − c2)2 +

1

2(ac+ bc− 2ab)2 − (ac+ bc− 2ab)(a2 − c2)

)≥ 0

²to je ekvivalentno s:

1

2

∑(a2 − c2 − ac− bc+ 2ab)2 ≥ 0

I nejednakost je dokazana. Jednakost o£ito vrijedi u slu£aju a = b = c, alirazlog zbog kojeg je ovaj zadatak toliko teºak je taj ²to jednakost vrijedijo² i u slu£aju kad su svi £lanovi zadnje sume jednaki 0, a to je, nakon maloraspisivanja, jo² i u slu£aju kad je:

a : b : c = sin2 4π

7: sin2 2π

7: sin2 π

7

Ovime je zadatak dovr²en.

*Napomena: Na sli£an na£in kao ²to je ovaj zadatak rije²en, moºe se doka-zati sljede¢i izuzetno jak teorem:

Teorem 5. Neka su a, b, c realni brojevi i n, p, q realne konstante. Tadanejednakost:

∑a4 + n

∑a2b2 + p

∑a3b+ q

∑ab3 − (1 + n+ p+ q)

∑a2bc ≥ 0

Vrijedi ako je 3 · (1 + n) ≥ p2 + pq + q2

Page 12: Kako zapisati rje²enja? - Math · Kako zapisati rje²enja? IMO zadaci i bodovne sheme ... Nemojte zaboraviti trivijalne slu£ajeve (nulu, prazan skup, bazu indukcije) Pi²ite obja²njenja,

Zadaci za vjeºbu

1. Neka su a, b, c, d > 0 realni brojevi takvi da je abcd = 1 i

a+ b+ c+ d >a

b+b

c+c

d+d

aDokaºite:

a+ b+ c+ d <b

a+c

b+d

c+a

d

2. Neka su a, b, c > 0 realni brojevi. Dokaºite da vrijedi:

(a)1

a(1 + b)+

1

b(1 + c)+

1

c(1 + a)≥ 3

1 + abc

(b)1

a(1 + b)+

1

b(1 + c)+

1

c(1 + a)≥ 3

3√abc(1 + 3

√abc)

3. Neka su a, b, c > 0 realni brojevi takvi da je a + b + c = 1. Dokaºite davrijedi:

ab√ab+ bc

+bc√

bc+ ca+

ca√ca+ ab

≤ 1√2

4. Neka su a, b, c > 0 realni brojevi takvi da je (a + b + c)(

1a

+ 1b

+ 1c

)= 10.

Odredite minimum i maksimum izraza:

(a2 + b2 + c2)

(1

a2+

1

b2+

1

c2

)5. Neka su a, b, c > 0, takvi da je a ≥ b ≥ c. Dokaºite da vrijedi:

√a(a−

√ab+ b) +

√b(b−

√bc+ c) +

√c(c−

√ca+ c) ≥ a+ b+ c

(a) uz uvjet da su a, b, c stranice trokuta(b) bez tog uvjeta

6. Neka su −1 < x1 < · · · < xn < 1 i y1 < · · · < yn realni brojevi takvi da je:

x1 + · · ·+ xn = x131 + . . . x13

n

Dokaºite da vrijedi:

x131 y1 + x13

2 y2 + · · ·+ x13n yn < x1y1 + x2y2 + · · ·+ xnyn

7. Neka su a, b, c realni brojevi, takvi da je a+ b+ c = 3. Dokaºi da je

(ab+ ac+ bc− 3)2 ≥ 27(abc− 1)

Kada vrijedi jednakost?

Page 13: Kako zapisati rje²enja? - Math · Kako zapisati rje²enja? IMO zadaci i bodovne sheme ... Nemojte zaboraviti trivijalne slu£ajeve (nulu, prazan skup, bazu indukcije) Pi²ite obja²njenja,

IMO pripreme 2015 Kristina Ana kreb, 14. 6. 2015.

Nestandardna algebra

Zadatak 1. Neka je P polinom stupnja n > 2 s cjelobrojnim koecijentimatakav da su mu nulto£ke razli£ite i u intervalu 〈0, 1〉. Neka je a vode¢i koecijentpolinoma P . Dokaºi da je |a| > 2n + 1.

Rje²enje: Neka su x1, x2, . . . , xn ∈ (0, 1) meusobno razli£ite nulto£ke polinoma

P (x) = a(x− x1)(x− x2) · · · (x− xn).

Primijetimo da je P (0)P (1) = (−1)n · a2∏n

i=1 [xi(1− xi)], odakle je

(−1)nP (0)P (1) = a2

n∏i=1

[xi(1− xi)].

Budu¢i da su nulto£ke polinoma P meusobno razli£ite, primjenom G-A nejed-nakosti dobivamo

(−1)nP (0)P (1) = a2

n∏i=1

[xi(1− xi)] < a2

n∏i=1

(1

2

)2

,

odakle je a2 > 22n · (−1)nP (0)P (1).

S obzirom na to da su koecijenti polinoma P cjelobrojni, vrijedi P (0), P (1) ∈Z \ 0.Ako je n paran, onda P (0) i P (1) imaju isti predznak; a ako je n neparan, P (0) iP (1) imaju razli£it predznak. Meutim, u oba slu£aja vrijedi (−1)nP (0)P (1) > 0,odnosno (−1)nP (0)P (1) ≥ 1 pa je kona£no a2 > 22n, odnosno |a| ≥ 2n + 1.

Zadatak 2. Neka su a1, a2, . . . , an i b1, b2, . . . , bn razli£iti realni brojevi. U ma-tricu n × n je na mjestu (i, j) napisan broj ai + bj. Ako znamo da je umnoºakelemenata u svakom retku jednak, dokaºite da je i umnoºak elemenata u svakomstupcu jednak.

Rje²enje: Umnoºak elemenata u i-tom retku iznosi

(ai + b1)(ai + b2) · · · (ai + bn)

i moºemo ga promatrati kao vrijednost polinoma

f(x) = (x+ b1)(x+ b2) · · · (x+ bn)

u to£ki x = ai. Iz uvjeta zadatka znamo da je

f(a1) = f(a2) = · · · = f(an) = c,

za neku konstantu c. To zna£i da su a1, a2, . . . , an nulto£ke polinoma f(x) − c.Budu¢i da je taj polinom stupnja n to su mu sve nulto£ke. Slijedi

(x+ b1)(x+ b2) · · · (x+ bn)− c = f(x)− c= (x− a1)(x− a2) · · · (x− an).

Page 14: Kako zapisati rje²enja? - Math · Kako zapisati rje²enja? IMO zadaci i bodovne sheme ... Nemojte zaboraviti trivijalne slu£ajeve (nulu, prazan skup, bazu indukcije) Pi²ite obja²njenja,

Ako sad uvrstimo x = −bj, dobit ¢emo da je

(−1)n(bj + a1)(bj + a2) · · · (bj + an) = f(−bj)− c = −c,

odnosno umnoºak elemenata u j-tom stupcu iznosi (−1)n+1c za svaki j = 1, 2, . . . , n.

Zadatak 3. Neka je f : (0, 1)→ (0, 1) funkcija denirana s

f(x) =

x+ 1

2ako x < 1

2,

x2 ako x > 12.

Neka su a i b realni brojevi takvi da je 0 < a < b < 1. Deniramo nizove an i bnrealnih brojeva na sljede¢i na£in:

a0 = a, b0 = b i an = f(an−1), bn = f(bn−1) za n > 0.

Dokaºite da postoji prirodan broj n takav da je

(an − an−1)(bn − bn−1) < 0.

Rje²enje: Uo£imo prvo da je

f(x)− x =1

2> 0 za x <

1

2,

if(x)− x = x2 − x < 0 za x >

1

2.

Stoga, ako interval (0, 1) podijelimo da dva disjunktna podintervala, I1 = (0, 12) i

I2 = [12, 1), nejednakost

(an − an−1)(bn − bn−1) =(f(an−1)− an−1

)(f(bn−1)− bn−1

)< 0

¢e vrijediti ako i samo ako an−1 i bn−1 leºe u razli£itim podintervalima (jedan uI1, a jedan u I2).

Pretpostavimo suprotno, tj. da ak i bk leºe uvijek u istom podintervalu. Ozna£imonjihovu udaljenost s dk (dk = |ak − bk|). Ako ak i bk oba leºe u I1, onda je

dk+1 = |ak+1 − bk+1| = |ak +1

2− bk −

1

2| = dk.

S druge strane, ako ak i bk oba leºe u I2 onda je min(ak, bk) > 12i max(ak, bk) =

min(ak, bk) + dk > 12

+ dk, pa iz toga slijedi

dk+1 = |ak+1−bk+1| = |a2k−b2

k| = |(ak−bk)(ak+bk)| > |ak−bk|(1

2+

1

2+dk

)= dk(1+dk) > dk.

Time smo dobili da je niz razlika dk neopadaju¢i, i posebno dk > d0 > 0 za svek. Moºemo dobiti i ja£e tvrdnje.

Ako ak i bk oba leºe u I2, onda vrijedi

dk+2 > dk+1 > dk(1 + dk) > dk(1 + d0).

Page 15: Kako zapisati rje²enja? - Math · Kako zapisati rje²enja? IMO zadaci i bodovne sheme ... Nemojte zaboraviti trivijalne slu£ajeve (nulu, prazan skup, bazu indukcije) Pi²ite obja²njenja,

Ako pak ak i bk oba leºe u I1, onda ak+1 i bk+1 oba leºe u I2, pa vrijedi

dk+2 > dk+1(1 + dk+1) > dk+1(1 + d0) = dk(1 + d0).

U oba slu£aja dobivamo dk+2 > dk(1 + d0), iz £ega induktivno slijedi

d2m > d0(1 + d0)2m.

Sada za dovoljno velike m vidimo da je desna strana gornje nejednakosti ve¢a od1, dok je s druge strane d2m < 1 (jer am i bm oba leºe u intervalu (0, 1)), ²to dajekontradikciju.

Time zaklju£ujemo da je na²a po£etna pretpostavka bila kriva, tj. postoji priro-dan broj n takav da an−1 i bn−1 ne leºe u istom podintervalu i stoga vrijedi

(an − an−1)(bn − bn−1) < 0.

Zadatak 4. Neka su z1, z2, . . . , zn kompleksni brojevi takvi da je

|z1|+ |z2|+ . . .+ |zn| = 1.

Dokaºi da postoji podskup S skupa z1, . . . , zn takav da je∣∣∣∣∣∑z∈S

z

∣∣∣∣∣ ≥ 1

6.

Rje²enje: Neka su l1, l2 i l3 tri polupravca iz ishodi²ta koji zatvaraju s pozitivnimdijelom x-osi kuteve od 60, 180 i 300 redom.

Za i = 1, 2, 3 ozna£imo sRi podru£je izmeu li i li+1 (ovdje je l4 = l1), uklju£uju¢ii polupravac li. Tada je

1 =∑zk∈R1

|zk|+∑zk∈R2

|zk|+∑zk∈R3

|zk|.

Prema Dirichletovom principu barem jedna od tri gornje sume je > 1/3. Nekaje to suma po R3 (ako nije ta, onda moºemo rotirati ²to ne mijenja vrijednostmodula kompleksnog broja). Neka je zk = xk + iyk. Tada za zk ∈ R3 vrijedixk = |xk| > |zk|/2. Slijedi∣∣∣ ∑

zk∈R3

zk

∣∣∣ > ∣∣∣ ∑zk∈R3

xk

∣∣∣ > 1

2

∑zk∈R3

|zk| >1

2· 1

3=

1

6.

Rje²enje: Dokazat ¢emo ja£u tvrdnju: postoji podskup S od z1, z2, . . . , zn takavda je ∣∣∣∣∣∑

z∈S

z

∣∣∣∣∣ ≥ 1

4.

Page 16: Kako zapisati rje²enja? - Math · Kako zapisati rje²enja? IMO zadaci i bodovne sheme ... Nemojte zaboraviti trivijalne slu£ajeve (nulu, prazan skup, bazu indukcije) Pi²ite obja²njenja,

Za k = 1, 2, . . . , n neka je zk = xk + iyk. Tada je

1 = |z1|+ |z2|+ · · ·+ |zn|6 (|x1|+ |y1|) + (|x2|+ |y2|) + · · ·+ (|xn|+ |yn|)

=∑xk>0

|xk|+∑xk<0

|xk|+∑yk>0

|yk|+∑yk<0

|yk|.

Prema Dirichletovom principu barem jedna od ove £etiri sume je > 1/4. Zbogsimetri£nosti moºemo pretpostaviti da je

1

46∑xk>0

|xk| =∣∣∣ ∑xk>0

xk

∣∣∣.Slijedi ∣∣∣ ∑

xk>0

zk

∣∣∣ > ∣∣∣ ∑xk>0

xk

∣∣∣ > 1

4.

Zadatak 5. Na plo£i su zapisani brojevi 1000, 1001, . . . , 2999. U svakom korakuBorna moºe obrisati dva broja, recimo a i b, te napisati broj 1

2mina, b.

Nakon 1999 takvih operacija na plo£i je ostao samo broj c. Dokaºi da je c < 1.

Rje²enje: Zbog simetri£nosti moºemo pretpostaviti da je a 6 b. Tada je

1

2min(a, b) =

a

2i

1

a+

1

b6

1(a2

) ,iz £ega slijedi da je suma recipro£nih vrijednosti brojeva na plo£i neopadaju¢a(tj. suma je monovarijanta). Ako sa S ozna£imo po£etnu sumu vrijedi

S =1

1000+

1

1001+ · · ·+ 1

29996

1

c.

Uo£imo da za 1 6 k 6 999 vrijedi

1

2000− k+

1

2000 + k=

4000

20002 − k2>

4000

20002=

1

1000,

pa je

1

c>

1

1000+( 1

1001+

1

2999

)+( 1

1002+

1

2998

)+ · · ·+

( 1

1999+

1

2001

)+

1

2000

>1

1000× 1000 +

1

2000> 1,

iz £ega slijedi da je c < 1.

Zadatak 6. Dano je n kompleksnih brojeva zk takvih da je |zk| ≤ 1, k =1, 2, . . . , n.

Dokaºi da postoje e1, . . . , en ∈ −1, 1 takvi da za svaki m ≤ n vrijedi

|e1z1 + e2z2 + . . .+ emzm| ≤ 2.

Page 17: Kako zapisati rje²enja? - Math · Kako zapisati rje²enja? IMO zadaci i bodovne sheme ... Nemojte zaboraviti trivijalne slu£ajeve (nulu, prazan skup, bazu indukcije) Pi²ite obja²njenja,

Rje²enje: Nazvat ¢emo kona£an niz kompleksnih brojeva zelenim ako su mu svielementi po modulu 6 1. Nazvat ¢emo zeleni niz zknk=1 sretnim ako postojiprijateljski niz eknk=1 kojemu su svi elementi 1 ili −1 koji zadovoljava uvjetezadatka.

Indukcijom po n ¢emo pokazati da su svi zeleni nizovi sretni.

Za n = 2 tvrdnja je o£ita.

Pretpostavimo da tvrdnja vrijedi za neki n i doka¢imo da vrijedi i za n + 1. Zasvaki k ozna£imo s lk pravac koji ide kroz ishodi²te i to£ku zk. Meu pravcimal1, l2, l3 postoje dva koja se sijeku pod kutem 6 60. Neka su to pravci lα i lβ, aonaj tre¢i ozna£imo s lγ. Budu£i da se lα i lβ sijeku pod kutem 6 60, slijedi dapostoji e′β ∈ −1, 1 takav da za z′ = zα + e′βzβ vrijedi |z′| 6 1.

Sada je niz z′, zγ, z4, . . . , zn+1 zeleni niz duljine n pa je prema pretpostavci in-dukcije sretan, tj. postoji prijateljski niz e′, eγ, e4, e5, . . . , en+1. Ako sad stavimoeα = e′ i eβ = e′e′β slijedi da je ekn+1

k=1 prijateljski niz od zkn+1k=1 , £ime je tvrdnja

dokazana.

Zadatak 7. Neka je (an)n niz realnih brojeva deniran s a1 = t i

an+1 = 4an(1− an), n ≥ 1.

Za koliko razli£itih vrijednosti broja t vrijedi a2015 = 0?

Rje²enje: Neka je f(x) = 4x(1− x). Uo£imo da je

f−1(0) = 0, 1, f−1(1) = 1/2, f−1([0, 1]) = [0, 1],

i |y : f(y) = x| = 2 za sve x ∈ [0, 1).

Neka je An = x ∈ R : fn(x) = 0. Tada je

An+1 = x ∈ R : fn+1(x) = 0= x ∈ R : fn(f(x)) = 0 = x ∈ R : f(x) ∈ An.

Tvrdimo da je za sve n > 1, An ⊂ [0, 1], 1 ∈ An i

|An| = 2n−1 + 1.

Za n = 1 imamoA1 = x ∈ R : f(x) = 0 = 0, 1,

i tvrdnja vrijedi. Pretpostavimo sad da je n > 1, An ⊂ [0, 1], 1 ∈ An i |An| =2n−1 + 1. Tada

x ∈ An+1 =⇒ f(x) ∈ An ⊂ [0, 1] =⇒ x ∈ [0, 1],

pa je i An+1 ⊂ [0, 1]. Budu¢i da je f(0) = f(1) = 0, imamo da je fn+1(1) = 0 zasve n > 1, pa e i 1 ∈ An+1. Sada imamo

|An+1| = |x : f(x) ∈ An| =∑a∈An

|x : f(x) = a|

= |x : f(x) = 1|+∑

a∈An,a∈[0,1)

|x : f(x) = a|

= 1 +∑

a∈An,a∈[0,1)

2 = 1 + 2(|An| − 1)

= 1 + 2(2n−1 + 1− 1) = 2n + 1,

Page 18: Kako zapisati rje²enja? - Math · Kako zapisati rje²enja? IMO zadaci i bodovne sheme ... Nemojte zaboraviti trivijalne slu£ajeve (nulu, prazan skup, bazu indukcije) Pi²ite obja²njenja,

£ime je tvrdnja dokazana.

Na kraju, a2015 = 0 ako i samo ako je f 2014(t) = 0, pa postoji 22013 + 1 razli£itihvrijednosti od t.

Rje²enje: Kao i u prethodnom rje²enju uo£imo da ako je f(x) ∈ [0, 1] mora bitix ∈ [0, 1], pa da bi imali a2015 = 0 mora vrijediti t ∈ [0, 1]. Izaberimo samθ ∈ [0, π/2] takav da je sin θ =

√t.

Uo£imo da za svaki φ ∈ R vrijedi

f(sin2 θ) = 4 sin2 θ(1− sin2 θ) = 4 sin2 θ cos2 θ = sin2 2θ.

Budu¢i da je a1 = sin2 θ, slijedi da je

a2 = sin2 2θ, a3 = sin2 4θ, . . . , a2015 = sin2 22014θ.

Stoga je

a2015 = 0 ⇐⇒ sin 22014θ = 0 ⇐⇒ θ =kπ

22014,

za neki k ∈ Z. Zna£i da su vrijednosti od t koje daju a2015 = 0 oblika sin2(kπ/22014),a takvih razli£itih ima 22013 + 1.

Zadatak 8. Dano je osam realnih brojeva a1, a2, . . . , a8 razli£itih od nule. Dokaºida je barem jedan od ²est brojeva a1a3+a2a4, a1a5+a2a6, a1a7+a2a8, a3a5+a4a6,a3a7 + a4a8, a5a7 + a6a8 nenegativan.

Rje²enje: Promotrimo vektore ~v1 = (a1, a2), ~v2 = (a3, a4), ~v3 = (a5, a6) i ~v4 =(a7, a8). Tada je

~v1 · ~v2 = a1a3 + a2a4, ~v1 · ~v3 = a1a5 + a2a6, ~v1 · ~v4 = a1a7 + a2a8

~v2 · ~v3 = a3a5 + a4a6, ~v2 · ~v4 = a3a7 + a4a8, ~v3 · ~v4 = a5a7 + a6a8.

Budu¢i da imamo £etiri vektora kad ih sve translatiramo da imaju po£etak uishodi²tu, vidimo da barem dva moraju zatvarati kut 6 90. Neka su to vektori~a i ~b. Iz ϕ = ∠(~a,~b) 6 90 slijedi cosϕ > 0, pa je

~a ·~b = |~a||~b| cosϕ > 0,

²to je i trebalo dokazati.

Zadatak 9. Neka je n > 2 cijeli broj i neka je f : R2 → R funkcija takva da zasvaki pravilan n-terokut A1A2 . . . An vrijedi

f(A1) + f(A2) + . . .+ f(An) = 0.

Dokaºi da je f nul-funkcija.

Rje²enje: Moºemo poistovjetiti R2 s kompleksnom ravninom. Ako ozna£imoξ = e2πi/n iz uvjeta zadatka slijedi da za svaki z ∈ C i pozitivan realan broj tvrijedi

n∑j=1

f(z + tξj) = 0.

Page 19: Kako zapisati rje²enja? - Math · Kako zapisati rje²enja? IMO zadaci i bodovne sheme ... Nemojte zaboraviti trivijalne slu£ajeve (nulu, prazan skup, bazu indukcije) Pi²ite obja²njenja,

Posebno, za svaki k = 1, 2, . . . , n dobijemo

n∑j=1

f(z − ξk + ξj) = 0,

²to zbrajanjem po svim k daje

n∑m=1

n∑k=1

f(z − (1− ξm)ξk) = 0.

Za m = n je unutarnja suma jednaka nf(z), a za druge m unutarnja suma opetide po vrhovima pravilnog mnogokuta i stoga je 0. Iz toga slijedi da je f(z) = 0za sve z ∈ C.

Zadatak 10. Za niz x1, x2, . . . , xn realnih brojeva deniramo njegovu cijenu kao

max16i6n

|x1 + · · ·+ xi|.

Za danih n realnih brojeva, Bobi i Rudi ih ºele poredati u niz sa ²to manjomcijenom. Rudi provjeri sve mogu¢e kombinacije i nae minimalnu mogu¢u cijenuR. Bobi s druge strane izabire x1 tako da je |x1| najmanji mogu¢i, zatim odpreostalih brojeva izabire x2 tako da je |x1 + x2| najmanje mogu¢e i tako daljeredom. Dakle, u i-tom koraku od preostalih brojeva izabere xi tako da je vrijed-nost |x1 +x2 + · · ·+xi| najmanja mogu¢a. Ako postoji vi²e kandidata za xi, Bobiizabere jednog na slu£ajan na£in. Na kraju dobije niz £ija je cijena B.

Naite najmanju konstantu c > 0 takvu da za svaki n ∈ N, svaki mogu¢i izborod n realnih brojeva i svaki mogu¢i niz koji Bobi moºe dobiti vrijedi

B 6 cR.

Rje²enje: Dokazat ¢emo da je c = 2.

Ako su dani brojevi 1,−1, 2,−2, Rudi ¢e ih poredati u niz 1,−2, 2,−1 i dobit¢e cijenu R = 1. S druge strane, Bobi sa svojom strategijom moºe dobiti niz1,−1, 2,−2 £ija je cijena B = 2. Dakle, c > 2.

Sada ¢emo dokazati da uvijek vrijedi B 6 2R. Neki su x1, x2, . . . , xn realni brojevidati Bobiju i Rudiju na raspolaganje. Pretpostavimo da su ih oni posloºili u nizoveb1, b2, . . . , bn i r1, r2, . . . , rn redom. Ozna£imo

M = max16i6n

|xi|, S = |x1 + · · ·+ xn|, i N = maxM,S.

Dokazat ¢emo da je

R > S, (1)

R >M

2, i (2)

B 6 N. (3)

Iz gornjih nejednakosti tada slijedi

B 6 maxM,S 6 maxM, 2S 6 2R.

Page 20: Kako zapisati rje²enja? - Math · Kako zapisati rje²enja? IMO zadaci i bodovne sheme ... Nemojte zaboraviti trivijalne slu£ajeve (nulu, prazan skup, bazu indukcije) Pi²ite obja²njenja,

Nejednakost (1) direktno slijedi iz denicije cijene niza.

Da bi dokazali (2) ozna£imo s i indeks (1 6 i 6 n) za koji je |ri| = M . Tada je

M = |ri| = |(r1 + · · ·+ri)−(r1 + · · ·+ri−1)| 6 |r1 + · · ·+ri|+ |r1 + · · ·+ri−1| 6 2R.

Preostaje samo dokazati nejednakost (3). Ozna£imo

si = b1 + b2 + · · ·+ bi.

Indukcijom ¢emo dokazati da za svaki i vrijedi |si| 6 N . Baza i = 1 direktnoslijedi, jer je |s1| = |b1| 6M 6 N . (Uo£imo da je i |sn| = S 6 N .)Za korak indukcije pretpostavimo da je |si−1| 6 N . Razlikujemo dva slu£aja:

(1) Svi brojevi bi, bi+1, . . . , bn su istog predznaka.BSOMP da su svi nenegativni. Tada je si−1 6 si 6 · · · 6 sn i stoga

|si| 6 max|si−1|, |sn| 6 N.

(2) Meu brojevima bi, bi+1, . . . , bn ima i pozitivnih i negativnih.Tada postoji indeks j > i takav da je si−1bj 6 0 (si−1 i bj su suprotnogpredznaka). Tada iz na£ina konstrukcije Bobijevog niza slijedi

|si| = |si−1 + bi| 6 |si−1 + bj| 6 max|si−1|, |bj| 6 N,

i time dokaz zavr²en.

Napomena 1. Moglo se i dokazati slabije nejednakosti

R >M

2i B 6 R +

M

2.

Zadatak 11. Neka je n ∈ N i a1, a2, . . . , an niz prirodnih brojeva. Periodi£nopro²irimo taj niz tako da deniramo an+i := ai za sve i > 1. Ako vrijedi

a1 6 a2 6 · · · 6 an 6 a1 + n (4)

iaai 6 n+ i− 1 za i = 1, 2, . . . , n, (5)

dokaºite da jea1 + · · ·+ an 6 n2.

Rje²enje: Prvo dokaºimo da je

ai 6 n+ i− 1 za i = 1, 2, . . . , n. (6)

Pretpostavimo suprotno, tj. da postoji neki indeks za koji gornja nejednakost nevrijedi i neka je i najmanji takav indeks. Iz

an > an−1 > · · · > ai > n+ i

Page 21: Kako zapisati rje²enja? - Math · Kako zapisati rje²enja? IMO zadaci i bodovne sheme ... Nemojte zaboraviti trivijalne slu£ajeve (nulu, prazan skup, bazu indukcije) Pi²ite obja²njenja,

i aai 6 n+ i− 1 zbog periodi£nosti niza slijedi da

ai 6≡ i, i+ 1, . . . , n− 1, n (mod n). (7)

Stoga iz pretpostavke ai > n + i slijedi da mora biti ai > 2n + 1, pa sad iz (4)imamo

a1 + n > an > a1 > 2n+ 1 =⇒ a1 > n+ 1.

Budu¢i da je i bio minimalan indeks za kojeg ne vrijedi (6), slijedi da je i = 1,pa dobivamo kontradikciju u (7). Time je (6) dokazana.

Sada znamo da je a1 6 n. Kad bi bilo i an 6 n imali bi

a1 6 a2 6 · · · 6 an 6 n,

pa bi traºena nejednakost slijedila trivijalno. U protivnome, neka je 1 6 t 6 n−1takav da je

a1 6 · · · 6 at 6 n < at+1 6 · · · 6 an. (8)

Budu¢i da je 1 6 a1 6 n i aa1 6 n iz (5), imamo a1 6 t i zato je an 6 n + t.Stoga ako za svaki prirodan broj i ozna£imo s bi broj indekasa j ∈ t+ 1, . . . , nza koje je aj > n+ i imamo

b1 > b2 > · · · > bt+1 = 0.

Sada tvrdimo da je ai + bi 6 n za 1 6 i 6 t. Zaista, zbog aai 6 n + i − 1 iai 6 n, svaki j koji zadovoljava aj > n + i (i zato aj > ai) se mora nalaziti uskupu ai + 1, . . . , n i zato je bi 6 n− ai.Iz denicije bi-ova i (8) slijedi da je

at+1 + · · ·+ an 6 n(n− t) + b1 + · · ·+ bt,

pa dodavanjem a1 + · · ·+ at na obje strane dobijemo

a1+· · ·+at+at+1+· · ·+an 6 n(n−t)+(a1+b1)+· · ·+(at+bt) 6 n(n−t)+nt = n2,

²to smo i ºeljeli dokazati.

Zadatak 12. Neka je P (x) polinom s realnim koecijentima takav da za svakadva realna broja x i y vrijedi

|y2 − P (x)| 6 2|x| ⇐⇒ |x2 − P (y)| 6 2|y|. (9)

Odredite sve mogu¢e vrijednosti od P (0).

Rje²enje: Dokazat ¢emo da su sve mogu¢e vrijednosti od P (0) sadrºane u (−∞, 0)∪1.

Prvo ¢emo dokazati da P (0) zaista moºe poprimiti sve vrijednosti iz skupa (−∞, 0)∪1.Za C > 0 deniramo polinom P (x) = −

(2x2

C+C

). Vidimo da je P (0) = −C < 0,

pa samo treba dokazati da taj polinom zadovoljava uvjet (9). To ¢emo napraviti

Page 22: Kako zapisati rje²enja? - Math · Kako zapisati rje²enja? IMO zadaci i bodovne sheme ... Nemojte zaboraviti trivijalne slu£ajeve (nulu, prazan skup, bazu indukcije) Pi²ite obja²njenja,

tako da pokaºemo da za svaka dva realna broja x i y vrijedi |y2 − P (x)| > 2|x|.Vrijedi

|y2 − P (x)| = y2 +x2

C+

(|x| − C)2

C+ 2|x| > x2

C+ 2|x| > 2|x|,

gdje prva nejednakost postaje jednakost za |x| = C, a druga za x = 0. Budu¢ida ne mogu oba uvjeta istovremeno biti ispunjena, slijedi |y2 − P (x)| > 2|x|.Preostalo je pokazati da je P (0) = 1 takoer mogu¢a vrijednost. Sad deniramopolinom P (x) = x2 + 1 i pokaºimo za zadovoljava (9). Za realne x i y imamo

|y2 − P (x)| 6 2|x| ⇐⇒ (y2 − x2 − 1)2 6 4x2

⇐⇒ 0 6(y2 − (x− 1)2

)((x+ 1)2 − y2

)⇐⇒ (y − x+ 1)(y + x− 1)(x+ 1− y)(x+ 1 + y)

⇐⇒((x+ y)2 − 1

)(1− (x− y)2

).

Budu¢i da je zadnja nejednakosti simetri£na u x i y, slijedi (9).

Sada ¢emo pokazati da su to stvarno jedine mogu¢e vrijednosti za P (0). Pret-postavimo da je P polinom koji zadovoljava (9) i za kojeg je P (0) > 0. Dokazat¢emo da je u tom slu£aju P (x) = x2 + 1 iz £ega naravno slijedi da je P (0) = 1.

1. Polinom P je paran.

Iz (9) slijedi da je

|y2 − P (x)| 6 2|x| ⇐⇒ |x2 − P (y)| 6 2|y| ⇐⇒ |y2 − P (−x)| 6 2|x|,

za sve realne x i y. Iz ekvivalencije prve i tre¢e nejednakosti slijedi[P (x)− 2|x|, P (x) + 2|x|

]∩ R>0 =

[P (−x)− 2|x|, P (−x) + 2|x|

]∩ R>0

za sve x ∈ R. Postoji beskona£no mnogo realnih brojeva x takvih da jeP (x) + 2|x| > 0. (To vrijedi jer je P (0) > 0. Moºemo pretpostaviti daje koecijent uz x u polinomu P nenegativan, pa ¢e traºena nejednakostvrijediti za sve dovoljno male pozitivne realne brojeve.)

Za x takve da P (x) + 2|x| > 0 tada vrijedi

P (x) + 2|x| = P (−x) + 2|x| =⇒ P (x) = P (−x).

To zna£i da polinom P (x) − P (−x) ima beskona£no mnogo nulto£aka, pamora biti identi£ki jednak nuli. Stoga je P (x) = P (−x) za svaki x ∈ R, tj.P je paran.

2. P (t) > 0 za sve t ∈ R.Pretpostavimo da postoji realan broj t 6= 0 takav da je P (t) = 0. Tadapostoji neki otvoreni interval I oko t takav da je |P (y)| 6 2|y| za sve y ∈ I.Uvr²tavanjem x = 0 u (9) dobijemo da je y2 = P (0) za sve y ∈ I, ²to jeo£ito nemogu¢e. Stoga je P (t) 6= 0 za sve t 6= 0.

Pretpostavimo sad da je P (0) = 0. Budu¢i da je P paran, postoji polinomQ takav da je P (x) = x2Q(x). Ako sad u (9) uvrstimo x = 0 i proizvoljany 6= 0 dobijemo da mora vrijedi |yQ(y)| > 2, ²to je o£ito nemogu¢e zadovoljno mali y.

Page 23: Kako zapisati rje²enja? - Math · Kako zapisati rje²enja? IMO zadaci i bodovne sheme ... Nemojte zaboraviti trivijalne slu£ajeve (nulu, prazan skup, bazu indukcije) Pi²ite obja²njenja,

3. P je kvadratni polinom.

Uo£imo da P ne moºe biti konstantan polinom, jer za x =√P (0) i dovoljno

veliki y je |y2−C| > 2|√P (0)| i |x2−P (y)| = |P (0)−P (y)| = 0 < 2|y|, pa

ne vrijedi (9). Ako ozna£imo stupanj polinoma P s n, mora vrijediti n > 2(jer je P paran).

Pretpostavimo sad da je n > 4. Uvr²tavanjem y =√P (x) u (9) dobivamo

|x2 − P (√P (x))| 6 2

√P (x), odnosno

P (√P (x)) 6 x2 + 2

√P (x)

za svaki x ∈ R. Izaberimo sad pozitivne realne brojeve x0, a i b takve da zax > x0 vrijedi

axn < P (x) < bx2.

(To je mogu¢e, jer je limx→∞P (x)xn

= d, gdje je d > 0 vode¢i koecijentpolinoma P .)

Sada za sve dovoljno velike x vrijedi

an/2+1xn2/2 < aP (x)n/2 < P (

√P (x)) 6 x2 + 2

√P (x) < xn/2 + 2b1/2xn/2,

odnosno

x(n2−n)/2 <1 + 2b1/2

an/2+1,

²to o£ito ne moºe biti istina. Stoga je P zaista kvadratni polinom.

4. P (x) = x2 + 1.

Budu¢i da je P paran, kvadratni polinom koji poprima pozitivne vrijednostiza sve realne brojeve, moºemo ga zapisati u obliku P (x) = ax2 + b, a > 0.Za dovoljno velik x i y =

√ax je |y2 − P (x)| = |b| 6 2|x|, pa zboj (9) mora

vrijediti i

|x2 − P (y)| 6 2|y| =⇒ |(1− a2)x2 − b| 6 2√ax.

Budu¢i da je a > 0 to moºe vrijediti samo ako je a = 1. Na kraju uvr²ta-vanjem y = x+ 1, x > 0 u (9) dobivamo

|2x+ 1− b| 6 2x ⇐⇒ |2x+ 1 + b| 6 2x+ 2,

odnosnob ∈ [1, 4x+ 1] ⇐⇒ b ∈ [−4x− 3, 1],

za sve x > 0. Ako izaberemo dovoljno velik x, moºemo posti¢i da je baremjedna od ove dvije tvrdnje istinita. Tada moraju obje biti istinite, a to jemogu¢e jedino za b = 1.

Page 24: Kako zapisati rje²enja? - Math · Kako zapisati rje²enja? IMO zadaci i bodovne sheme ... Nemojte zaboraviti trivijalne slu£ajeve (nulu, prazan skup, bazu indukcije) Pi²ite obja²njenja,

IMO pripreme 2015. Nikola Adºaga, 16. 6. 2015.

Kombinatorna teorija brojeva

Uvod

U ovom predavanju pro¢i ¢emo razne zadatke koji zahtjevaju poznavanje i kom-binatorike i teorije brojeva. Iako je to jedino zajedni£ko svim zadacima, dio njihspadat ¢e u dva bliska podru£ja aditivna kombinatorika i aditivna teorija bro-jeva.

Aditivna teorija brojeva neki rezultati

Jedan klasi£ni poznati teorem je Cauchy-Davenport, koji se moºe iskazati nasljede¢i na£in: Ako je t prirodan broj i x1, x2, . . . , xt ne-nul elementi od Zp (ostacimodulo p) koji nisu nuºno razli£iti. Tada se barem minp, t + 1 elemenata odZp da napisati kao zbroj brojeva xi .

Pomo¢u tog teorema da se dokazati Erd®s-Ginzburg-Ziv teorem svaki skup od2n− 1 cijelih brojeva sadrºi n-£lani podskup £iji je zbroj elemenata djeljiv s n.

Od novijih rezultata poznat je Green-Tao teorem: skup prostih brojeva sadrºiproizvoljno duge aritmeti£ke nizove.

Aditivna kombinatorika

Bavi se op¢enitijim problemima, naj£e²¢e vezanim za prisutnost aritmeti£kih ni-zova u nekim skupovima. Poznat je rezultat Rothov teorem za svaki pozitivanδ 6 1 postoji prirodan broj N = N(δ) takav da svaki podskup od 1, 2, . . . , Nveli£ine barem δN sadrºi tro£lani aritmeti£ki niz.

To je dobar predstavnik jednog tipa problema u aditivnoj kombinatorici kolikoveliki moºe biti podskup nekog ksnog skupa bez da sadrºi odreenu strukturu?

Druga vrsta problema su inverzni problemi. Ako je A skup prirodnih brojevatakav da je |A+A| 6 3|A|, ²to moºemo re¢i o A? Za neki slu£ajno odabran skupA o£ekivali bismo da je veli£ina od A+ A otprilike |A|2, ali ako je A aritmeti£kiniz, znamo da je |A + A| = 2|A| − 1. Mora li A za koji je |A + A| 6 3|A| bitiaritmeti£ki niz?

Ne¢emo ulaziti u dokaze ovih teorema (iako neki imaju elementarne dokaze) sve ²to treba zapamtiti odavdje je da ima smisla traºiti aritmeti£ke nizove uodreenim skupovima, ili npr. trojke takve da je a+ b > c, i sl.

Page 25: Kako zapisati rje²enja? - Math · Kako zapisati rje²enja? IMO zadaci i bodovne sheme ... Nemojte zaboraviti trivijalne slu£ajeve (nulu, prazan skup, bazu indukcije) Pi²ite obja²njenja,

Zadaci

1. Za skup A, neka |A| i s(A) redom ozna£avaju broj £lanova A i sumu £lanovaiz A.Neka je S skup prirodnih brojeva takav da:

(a) postoje dva broja x, y ∈ S takvi da im je mjera 1

(b) za bilo koja dva broja x, y ∈ S, x+ y ∈ S.

Neka je T = N \ S. Dokaºite s(T ) ≤ |T |2 <∞.(prijedlog za USAMO 2000., 102 Combinatorial Problems).

2. Dan je skup A = 1, 2, 3, . . . , 2n− 1. Obri²i barem n− 1 broj iz A premasljede¢im pravilima:

• Ako je broj a izbrisan te je 2a ∈ A, onda mora² izbrisati i broj 2a.

• Ako su brojevi a i b izbrisani te je a + b ∈ A, onda mora² izbrisati ibroj a+ b.

Neka je S zbroj obrisanih brojeva. Odredi minimalnu mogu¢u vrijednostod S.

(Vijetnamska olimpijada 1990.)

3. Neka je A skup prvih 16 prirodnih brojeva. Nai najmanji prirodan brojk sa sljede¢im svojstvom: U svakom podskupu od A s k elemenata postojedva razli£ita broja a i b takvi da je a2 + b2 prost.

(Vijetnamska olimpijada 2004.)

4. Neka je S skup od 2004 prirodna broja. Za svaki a ∈ S neka je f(a) brojbrojeva u S koji su relativno prosti s a. Pretpostavimo da je f(a) < 2003isti za sve a ∈ S. Nai najmanji k takav da svaki podskup od k elemenataod S sadrºi dva broja koja nisu meusobno relativno prosta.

(Viet Nam TST 2004.)

5. Neka je X kona£an skup prirodnih brojeva, A ⊆ X. Dokaºite da postojiB ⊆ X takav da je A jednak skupu brojeva iz X koji dijele neparno mnogo£lanova B.

6. Na po£etku igre za 2 igra£a (igraju naizmjence), broj 2015! napisan je naplo£i. U svakom potezu, igra£ izabere prirodni broj s najvi²e 20 razli£itihprostih faktora, koji je manji ili jednak od onog na plo£i. Broju na plo£ioduzme se taj (izabrani) broj. Pobjednik je igra£ koji napi²e 0. Koji igra£sigurno moºe pobijediti, i kako? (Turnir gradova, prolje¢e 2004., Senior A)

7. Prije po£etka igre, Ivan zamisli prirodan broj ve¢i od 100. Marija zatim biraprirodan broj d > 1. Ako je Ivanov broj djeljiv s d, Marija pobjeuje. Usuprotnom, Ivan oduzima d i igra se nastavlja (s novim brojem). Marija nesmije ponoviti broj. Kad Ivanov broj postane negativan, Marija je izgubila.Postoji li pobjedni£ka strategija za Mariju? (Turnir gradova, prolje¢e 2003.,Senior A)

Page 26: Kako zapisati rje²enja? - Math · Kako zapisati rje²enja? IMO zadaci i bodovne sheme ... Nemojte zaboraviti trivijalne slu£ajeve (nulu, prazan skup, bazu indukcije) Pi²ite obja²njenja,

8. Rastu¢i niz 1, 3, 4, 9, 10, 12, 13, . . . sadrºi sve prirodne brojeve koji su poten-cija broja 3 ili suma razli£itih potencija broja 3. Naite 100. £lan tog niza.(AIME 1986.)

9. Neka je A skup prirodnih brojeva takav da je |A| = 2001. Dokaºite dapostoji skup B takav da vrijedi

• B ⊆ A

• |B| > 668

• ∀u, v ∈ B (ne nuºno razli£ite), u+ v /∈ B.

(USA TST 2001.)

10. Neka je p neparan prost broj. Naite broj svih podskupova A skupa1, 2, . . . , 2p tako da

• A ima to£no p elemenata, i

• zbroj svih elemenata skupa A je djeljiv s p.

(IMO 1995./6.)

Zadaci za samostalan rad

1. Odredite najmanji prirodni broj n > 4, za koji je izmeu svakih n razli£itihprirodnih brojeva mogu¢e odabrati 4 razli£ita a, b, c, d tako da je a+b−c−ddjeljivo s 20. IMO Shortlist 1998., N2

2. Neka je p > 2 prost broj. Promotrimo poligon s p bridova s majmunomna svakom vrhu. Daj p banana majmunima po sljede¢im pravilima. Prvudaj bilo kojem. Zatim odi tri vrha u smjeru kazaljke na satu i drugu dajmajmunu na tom vrhu, . . ., (k+ 1). bananu daj majmunu na (2k+ 1). vrhudalje od majmuna s k-tom bananom (u istom smjeru).

• Koliko majmuna ne dobije nijednu bananu?

• Koliko bridova poligona ima svojstvo da su majmuni na oba njegovakraja dobili banane?

Viet Nam TST 1999.

3. Dana su dva prirodna broja m,n > 1 gdje m nije djeljiv s n. Nai najmanjiprirodni k > 1 sa svojstvom da za k proizvoljnih cijelih a1, a2, . . . , ak takvihda za svaki izbor 1 6 i < j 6 k broj ai − aj nije djeljiv s n, postoje dvabroja as, at(s 6= t) meu njima takvi da je as − at +m djeljivo s n.

(Viet Nam TST 1992.)

Dodatni izvori zadataka

• Matija Ba²i¢, Brojevne baze, http://web.math.pmf.unizg.hr/ mbasic/materijali.htm

• coach.rar

Page 27: Kako zapisati rje²enja? - Math · Kako zapisati rje²enja? IMO zadaci i bodovne sheme ... Nemojte zaboraviti trivijalne slu£ajeve (nulu, prazan skup, bazu indukcije) Pi²ite obja²njenja,

IMO pripreme 2015. Ivan Krijan, 15. 06. 2015.

Linearna algebra u kombinatorici

Osnovni pojmovi i rezultati linearne algebre

Prvo ²to nam je potrebno je znati ²to je to uop¢e polje? Osnovne primjere, jasno,znamo, npr. Q, R, C. Navedimo op¢enitu definiciju:

Polje je neprazan skup F s operacijama + : F×F→ F (zbrajanje) i · : F×F→ F(mnoºenje) takvima da je:

• (asocijativnost operacija), a + (b + c) = (a + b) + c i a · (b · c) = (a · b) · c,za sve a, b, c iz F.

• (komutativnost operacija), a+ b = b+ a i a · b = b · a, za sve a, b ∈ F.

• (neutralni elementi), postoje 0 ∈ F i 1 ∈ F takvi da je 0 + a = a i 1 · a = a,za sve a ∈ F.

• (inverzi), za svaki a ∈ F postoji −a ∈ F takav da je a+ (−a) = 0, takoer,za svaki a ∈ F, a 6= 0, postoji a−1 ∈ F takav da je a · a−1 = 1.

• (distributivnost mnoºenja prema zbrajanju), a · (b+ c) = a · b+ a · c, za svea, b, c, ∈ F.

Primjer 1. (zadatak za zagrijavanje) Neka je F polje. Dokaºite da su 0, 1, −a, a−1

jedinstveni. Ako je |F| > 1, dokaºite da je 0 6= 1.

Osim klasi£nih primjera polja, ono ²to se £esto pojavljuje su polja

Zp = 0, 1, . . . , p− 1 ,

uz operacije zbrajanja i mnoºenja modulo p. Ovdje nam je p, jasno, prost broj.Moºete li dokazati da je ovo polje?

Od glavnog interesa (kao ²to ¢emo i vidjeti kroz zadatke) ¢e nam biti polje Z2.Nije na odmet posebno napomenuti: Svi ovdje navedeni rezultati vrijede zaslu£aj bilo kojeg polja F!Idu¢i korak (prije hrvanja sa zadacima) nam je uvesti (osnovne) pojmove line-arne algebre, kao ²to su: vektor, linearna (ne)zavisnot, skalarni produkt,matrica, rang, determinanta, itd.

Iako se moºe op¢enito definirati, nama to ovdje nije potrebno pa ¢emo se drºatijednostavnijeg prikaza.

Za n ∈ N ¢emo s Fn ozna£avati vektorski prostor svih ureenih n-torki ele-menata iz F koje ¢emo zvati vektori. Zbrajanje i mnoºenje skalarom vektora¢emo denirati po koordinatama. Preciznije, za vektore a = (a1, a2, . . . , an) ib = (b1, b2, . . . , bn) te skalar α ∈ F imamo:

a+ b = (a1 + b1, a2 + b2, . . . , an + bn) i αa = (αa1, αa2, . . . , αan).

Page 28: Kako zapisati rje²enja? - Math · Kako zapisati rje²enja? IMO zadaci i bodovne sheme ... Nemojte zaboraviti trivijalne slu£ajeve (nulu, prazan skup, bazu indukcije) Pi²ite obja²njenja,

Za vektore a1, a2, . . . , am ∈ Fn ¢emo re¢i da su linearno nezavisni ako vrijedi:Ako su α1, α2, . . . , αm skalari takvi da je

α1a1 + α2a2 + . . .+ αmam = 0,

onda je α1 = α2 = . . . = αm = 0. U suprotnom ¢emo re¢i da su linearno zavisni.

Za vektore a1, a2, . . . am ∈ Fn, ili op¢enito za podskup S ⊆ Fn ¢emo sa

〈a1, a2, . . . am〉 , odnosno sa 〈S〉 ,

ozna£avati potprostor razapet sa skupom S. To£nije, 〈S〉 se sastoji od svih mogu-¢ih linearnih kombinacija vektora iz S. Linearna kombinacija vektora je sumatih vektora pomnoºenih sa skalarima.

Baza vektorskog prostora je linearno nezavisni sistem izvodnica. Skup S ⊆ Fnje sistem izvodnica za Fn, ako je 〈S〉 = Fn. Dimenzija vektorskog prostora jebroj elemenata njegove baze.

Primijetimo da definicija dimenzije ima smisla jedino ako znamo da svake dvijebaze vektorskog prostora imaju jednak broj elemenata. Na svu sre¢u, to je to£no!

Teorem 6.

• Dimenzija vektorskog prostora Fn je n.

• Svaki linearno nezavisan skup vektora u Fn sadrºi ≤ n elemenata. Svakisistem izvodnica za Fn sadrºi ≥ n elemenata.

Napomenimo jo² i sljede¢e: Ako je F kona£no polje onda je broj elemenatasvakog potprostora V od Fn jednak |F|dimV .

Primjer 2. Pokaºite da se svaki vektor moºe na jedinstven na£in zapisati u bazi.

Ova na²a pri£a o bazi vektorskog prostora je za kona£no dimenzionalne vektorskeprostore. Ono ²to je zanimljivo je da baza postoji £ak i ako je vektorski prostorbeskona£no dimenzionalan.

Hamelova baza vektorskog prostora R nad poljem Q je skup realnih brojeva(ri)i∈I koji je linearno nezavisan (svaki kona£an podskup mu je linearno nezavi-san) i takav da se svaki x ∈ R moºe prikazati u obliku:

x =n∑k=1

qkrk,

gdje je n ∈ N koji ovisi o x, qk su racionalni brojevi, a rk su elementi baze. Dakle,imajmo na umu da znamo da Hamelova baza za R postoji!

Primjer 3. Neka je u krug rasporeeno n ≥ 3 realnih brojeva tako da za svaka triuzastopna vrijedi da je jedan od njih jednak aritmeti£koj sredini preostala dva.Dokaºite da su ili svi brojevi meusobno jednaki ili je n djeljiv s 3.

Prije nego uvedemo idu¢e objekte (matrice) denirajmo jo² skalarni produkt vek-tora. Opet, ovo se moºe napraviti puno op¢enitije, ali za na²e potrebe je idu¢e ivi²e nego dovoljno.

Page 29: Kako zapisati rje²enja? - Math · Kako zapisati rje²enja? IMO zadaci i bodovne sheme ... Nemojte zaboraviti trivijalne slu£ajeve (nulu, prazan skup, bazu indukcije) Pi²ite obja²njenja,

Skalarni produkt dvaju vektora a, b ∈ Fn definiramo kao:

a · b =n∑k=1

akbk,

ovdje nam je bk oznaka za kompleksni konjugat broja bk. Jasno, kako ¢emo se mibaviti samo realni prostorima, to nam ne¢e predstavljati nikakvu ulogu.

Skalarni produkt je funkcija koja svakom paru vektora pridruºi skalar. Popi²imoosnovna svojstva skalarnog produkta:

• a · a ≥ 0 i a · a = 0, ako i samo ako je a = 0,

• b · a = a · b,

• (αa+ βb) · c = αa · c+ βb · c.

Zanimljivost! Ako je V vektorski prostor nad poljem F i imamo li preslikavanje〈·, ·〉 : V × V → V koje zadovoljava gore popisana svojstva skalarnog produkta,onda vrijedi nejednakost CauchySchwarzBunyakovsky:

| 〈a, b〉 |2 ≤ 〈a, a〉 〈b, b〉 .

Primjer 4. (St. Petersburg) U£enici u grupama od po barem dvoje odlaze posladoled. Nakon ²to je k > 1 grupa oti²lo, svaka dva studenta su oti²la zajednoto£no jednom. Dokaºite da je broj studenata najvi²e jednak k.

Matrica m×n je naprosto tablica s m redaka i n stupaca koja na svakom mjestuima upisan neki skalar iz F.

A = (ai,j)i=1, 2, ...,m, j=1, 2, ..., n =

a1,1 a1,2 · · · a1,n

a2,1 a2,2 · · · a2,n...

... . . . ...am,1 am,2 · · · am,n

Matrice zbrajamo jednostavo po koordinatama, tako ih takoer i mnoºimo saskalarima. Mnoºenje matrica je ipak ne²to kompliciraniji postupak.

Neka su A i B, redom m× n i n× k matrice, tada je produkt AB = C matricatipa m× k. Dakle, da bi imalo smisla mnoºiti matrice one moraju biti ulan£ane,tj. broj stupaca prve, mora biti jednak broju stupaca druge. Moramo jo² definirati²to je uop¢e matrica C?

cij =n∑l=1

ailblj.

Napomenimo da, najprije, umnoºak BA ne mora uop¢e biti definiran, a ako je,tada ne mora vrijediti da je AB = BA. Dakle, op¢enito je AB 6= BA. Znateli navesti primjer nekih matrica koje ne komutiraju? (ali da je mnoºenje dobrodefinirano u oba poretka)

Prostor svih matrica m × n ozna£avamo s Mmn(F). Dimenzija tog prostora jemn.

Page 30: Kako zapisati rje²enja? - Math · Kako zapisati rje²enja? IMO zadaci i bodovne sheme ... Nemojte zaboraviti trivijalne slu£ajeve (nulu, prazan skup, bazu indukcije) Pi²ite obja²njenja,

Za matricu A ∈Mmn, s At ozna£avamo transponiranu matricu, to je matricatipa n×m £iji su koeficijenti jednaki aji.

Rang matrice je broj linearno nezavisnih stupaca matrice. Taj broj je uvijek jed-nak broju linearno nezavisnih redaka! Za matricu A, njen rang ¢emo ozna£avatis r(A). Osnovno o rangu, za matrice A, B ∈Mm,n i C ∈Mn,k:

• r(A) ≤ min(m,n),

• r(A+B) ≤ r(A) + r(B),

• r(AC) ≤ min(r(A), r(C)).

Promotrimo sustav jednadºbi:

a1,1x1 + a1,2x2 + . . .+ a1,nxn = b1

a2,1x1 + a2,2x2 + . . .+ a2,nxn = b2

...am,1x1 + am,2x2 + . . .+ am,nxn = bm

Stavimo li da je A = (ai,j) vidimo da taj sustav moºemo zapisati kao:

Ax = b,

ovdje su x i b vektor stupci, tj. matrice tipa m× 1.

Teorem 7 (Kronecker Capelli). Gornji sustav ima rje²enje ako i samo ako jer(A) = r(Ap) gdje je matrica Ap (pro²irena) nastala od matrice A dodavanjemstupca b.

Pretpostavimo da gornji sustav ima rje²enja. Znamo da ih on tada ima ili jednoili beskona£no (ako smo nad beskona£nim poljem). No, svako rje²enje sustavamoºemo zapisati u obliku

x1 + x0,

gdje je x1 bilo koje rje²enje (partikularno), a x0 bilo koje rje²enje pripadne ho-mogene (kada je b = 0) jednadºbe.

Defekt matrice A je dimenzija prostora svih rje²enja jednadºbe Ax = 0, oznakaje d(A).

Teorem 8 (o rangu i defektu). Ako je A ∈Mm,n, onda je r(A) + d(A) = n.

Primjer 5. (VJIMC, autor: Vjekoslav Kova£) Neka su k i n prirodni brojevitakvi da je k ≤ n−1. Neka je S = 1, 2, . . . , n i neka su A1, A2, . . . , Ak nepraznipodskupovi skupa S. Dokaºite da je mogu¢e obojiti neke elemente od S u dvijeboje, crvenu i plavu, tako da vrijede sljede¢i uvjeti:

1. Svaki element od S je neobojen ili je obojen u crveno ili plavo.

2. Barem jedan element skupa S je obojen.

3. Svaki skup Ai (i = 1, 2, . . . , k) je ili £itav neobojen ili je barem jedan elementobojen crveno i barem jedan element obojen plavo.

Page 31: Kako zapisati rje²enja? - Math · Kako zapisati rje²enja? IMO zadaci i bodovne sheme ... Nemojte zaboraviti trivijalne slu£ajeve (nulu, prazan skup, bazu indukcije) Pi²ite obja²njenja,

Ovdje stajemo s op¢enitim matrica i prelazimo na one koje ¢e nama biti zanim-ljive. To su kvadratne matrice, tj. one tipa n×n. Prostor tih matrica se ozna£avajednostavno Mn, a ne Mn,n. U prostoru kvadratnh matrica postoji ne²to ²to na-zivamo identiteta ili jedini£na matrica. To je matrica I = (δi,j), gdje je δi,jKroneckerova delta:

δi,j =

1, i = j,0, i 6= j.

Za identitetu vrijedi da je AI = IA = A, za sve A ∈ Mn. Za matricu A ∈ Mn

kaºemo da je regularna ili invertibilna ako postoji matrica B ∈ Mn takva daje AB = BA = I. Takvu matricu B onda ozna£avamo s A−1. Matricu koja nijeregularna nazivamo singularna.

Teorem 9. Matrica A ∈Mn je regularna ako i samo ako ima potpun rang, tj. akoje r(A) = n.

Primijetimo da je matrica A regularna ako i samo ako je d(A) = 0, a to zna£i daje regularna ako i samo ako ne postoji netrivijalan vektor x takav da je Ax = 0.

Rang se (na svu sre¢u) ne mora uvijek ra£unati direktno po definiciji (tj. skoronikad ga ne¢ete tako ra£unati). Naime, postoji ne²to ²to se zove elementarnetransformacije matrice. One su:

1. dodavanje jednog retka (stupca), pomnoºenog nekim skalarom, nekom dru-gom retku (stupcu), to£nije: uzmemo li redak (stupac) i i redak (stupac)j (i 6= j), tada je rang matrice A isti kao rang matrice u kojoj smo redak(stupac) j zamijenili retkom (stupcom) j + αi,

2. zamjena bilo koja dva retka ili stupca matrice A,

3. mnoºenje jednog retka (stupca) sa skalarnom 6= 0.

Kori²tenjem elementarnih transformacija moºemo matricu lako svesti na ne²to²to se zove gornje trokutasta matrica (svi elementi ispod glavne dijagonale(i = j) su jednaki 0). Jasno je da je takvim matricama lako o£itati rang.

Uvedimo jo² jedan pojam koji ¢e nam trebati prije no ²to (NAPOKON) prijeemona toliko is£ekivane zadatke.

Za kvadratnu matricu A ∈Mn definiramo njenu determinantu:

detA =∑σ∈Sn

(−1)sgnσa1σ(1)a2σ(2) · · · anσ(n).

Ovdje je sgnσ parnost permutacije σ. Definira se kao broj transpozicija upermutaciji σ, pogledamo taj broj modulo 2. Transpozicija je svaki par indeksai < j takav da je σ(i) > σ(j). Vrijedi da je: detAt = detA.

Teorem 10 (Binet Cauchy). det(AB) = detA detB.

Primijetimo: Ako nam je polje Z2, u tom polju vrijedi −1 = 1 pa nam onda udefiniciji determinante zapravo nema £lana (−1)sgnσ.

Determinantu matrice takoer ne moramo (iako je nekada zgodno) ra£unati podefiniciji. Najprije, postoji ne²to ²to se zove Laplaceov razvoj po retku, odnosno

Page 32: Kako zapisati rje²enja? - Math · Kako zapisati rje²enja? IMO zadaci i bodovne sheme ... Nemojte zaboraviti trivijalne slu£ajeve (nulu, prazan skup, bazu indukcije) Pi²ite obja²njenja,

po stupcu:Za svaki i vrijedi da je:

detA =n∑j=1

(−1)i+jaij detAij,

ovdje je Aij matrica dobivena iz matrice A izbacivanjem retka i i stupca j. Istaformula vrijedi ksirao li j i sumiramo po i.

Determinanta se takoer dobro pona²a pri elementarnim transformacijama. Pritransformaciji 1 determinanta se ne mijenja. Pri transformaciji 2 determinantamijenja predznak, dok pri transformaciji 3 se cijela determinant pomnoºi timskalarom. Sva ova pravila se vide direktno iz definicije determinante.

Takoer ako je matrica gornje ili donje trokutasta, tada je njena determinantajednaka umno²ku dijagonalnih elemenata (²to se opet lako vidi iz definicije). Lakoizr£aunamo da je

det

(a bc d

)= ad− bc.

Teorem 11. Matrica A ∈Mn je regularna ako i samo ako je detA 6= 0.

Primjer 6. Izra£unajte determinantu n× n matrice

2 3 0 0 · · · 0 01 2 3 0 · · · 0 00 1 2 3 · · · 0 00 0 1 2 · · · 0 0...

......

... . . . ......

0 0 0 0 · · · 2 30 0 0 0 · · · 1 2

.

Primjer 7. Neka je n > 1 prirodan broj i neka je, za σ ∈ Sn, I(σ) = 1 ako jeσ parna permutacija, a I(σ) = −1 ako je σ neparna permutacija. Neka je f(σ)broj fiksnih to£aka permutacije σ. Odredite sva rje²enja jednadºbe∑

σ∈Sn

I(σ)xf(σ) = 0,

u skupu pozitivnih realnih brojeva.

Za vektore a, b ∈ Fn kaºemo da su ortogonalni ako je a · b = 0. Ako je S ⊆ Fndefiniramo njegov ortogonalni komplement kao skup svih vektora okomitih na svevektore uz S. Oznaka je S⊥.

injenica je da je S⊥ = 〈S〉⊥, takoer S⊥ je uvijek potprostor od Fn. Vrijedi daje:

dim 〈S〉⊥ = dimFn − dim 〈S〉 = n− dim 〈S〉 .

Primjer 8. Neka su A1, A2, . . . , Am razli£iti podskupovi od 1, 2, . . . , n takvida je |Ai ∩ Aj| paran broj za sve i 6= j.

(a) Ako je |Ai| paran broj za sve i, koliko najvi²e moºe biti m, u odnosu na n?

(b) Ako je |Ai| neparan broj za sve i, isto pitanje?

Page 33: Kako zapisati rje²enja? - Math · Kako zapisati rje²enja? IMO zadaci i bodovne sheme ... Nemojte zaboraviti trivijalne slu£ajeve (nulu, prazan skup, bazu indukcije) Pi²ite obja²njenja,

Zadaci

1. Odredite najmanji prirodni broj n sa svojstvom: Ako su x1, x2, x3, x4, x5

realni brojevi takvi da postoji n razli£itih odabira cijelih brojeva 1 ≤ p <q < r ≤ 5 takvih da je xp+xq+xr = 0, onda je x1 = x2 = x3 = x4 = x5 = 0.

2. (Fisherova nejednakost, primjer 4) Neka su A1, A2, . . . , Am razli£iti pod-skupovi skupa 1, 2, . . . , n. Pretpostavimo da postoji cijeli broj 1 ≤ λ ≤ ntakav da je |Ai ∩ Aj| = λ, za sve i 6= j. Tada je m ≤ n.

3. Dan je prirodan broj n i n-£lani skup S. Neka je

A = A1, A2, . . . , An

familija koja se sastoji od n meusobno razli£itih podskupova skupa S.Dokaºite da postoji x ∈ S takav da su svi skupovi

A1 ∪ x, A2 ∪ x, . . . , An ∪ x

meusobno razli£iti.

4. (Iran TST 1996, Germany TST 2004) Neka je G kona£an i jednostavangraf kojem je svaki vrh obojen u bijelo. U svakom koraku dozvoljeno jeodabrati vrh i promjeniti boju (bijelo u crno ili crno u bijelo) tom vrhu isvim njegovim susjedima. Dokaºite da je mogu¢e posti¢i da su svi vrhovicrni.

5. (Gallaijev teorem) Skup vrhova svakog jednostavnog grafa se moºe partici-onirati u dva skupa (ne nuºno neprazna) tako da svaki skup vrhova inducirapodgraf u kojemu su svi vrhovi parnog stupnja. Dokaºite.

6. U senatu je 2015 senatora. Svaki senator ima neprijatelje unutar senata (poprincipu, ako si ti neprijatelj meni, onda sam bome i ja tebi). Dokaºite dapostoji neprazan podskup K skupa svih senatora takav da svaki senator usenatu ima paran broj neprijatelja u skupu K.

7. (Moldova TST 2005) Postoji li konguracija od 22 razli£ite kruºnice i 22razli£ite to£ke u ravnini takve da svaka kruºnica sadrºi barem 7 to£aka isvaka to£ka pripada barem 7 kruºnica?

8. Neka su a1, a2, . . . , a2n+1 realni brojevi takvi da, za svaki 1 ≤ i ≤ 2n + 1,moºemo maknuti ai te ostalih 2n brojeva podijeliti u dvije grupe od nbrojeva £ija je suma jednaka. Dokaºite da je a1 = a2 = . . . = a2n+1.

9. Dokaºite: ako su svi kostupnjevi (kostupanj para vrhova je broj vrhova kojisu incidentni s oba vrha) jednostavnog grafa s n vrhova neparni, onda je nneparan.

10. Na tulumu se nalazi 2n ljudi. Svaka osoba ima paran broj prijatelja (prija-teljstvo smatramo simetri£nom relacijom). Dokaºite da postoje dvije osobekoje imaju paran broj zajedni£kih prijatelja na tulumu.

Page 34: Kako zapisati rje²enja? - Math · Kako zapisati rje²enja? IMO zadaci i bodovne sheme ... Nemojte zaboraviti trivijalne slu£ajeve (nulu, prazan skup, bazu indukcije) Pi²ite obja²njenja,

11. (sli£no kao prethodni zadatak, ali nije isto, za²to?) Za skup T kaºemo daje paran ako ima paran broj elemenata. Neka je n paran prirodan broj ineka su S1, S2, . . . , Sn parni podskupovi skupa 〈1, 2, . . . , n〉. Dokaºite dapostoje neki i 6= j takvi da je |Si ∩ Sj| paran broj.

12. Imamo n nov£i¢a nepoznatih masa i ravnoteºnu vagu s dvije zdjelice. Smi-jemo staviti nekoliko nov£i¢a na jednu zdjelicu i isti broj nov£i¢a na drugu.Ono ²to moºemo nakon toga o£itati na vagi je koja je zdjelica teºa ili supak jednake. Pokaºite da je potrebno barem n− 1 takvih vaganja da bismomogli zaklju£iti da su svi nov£i¢i jednake mase.

13. (USAMO 2008, problem 6) Na matemati£koj konferenciji, svaka dva ma-temati£ara su ili prijatelji ili stranci. U vrijeme ru£ka, svaki matemati£arru£a u jednoj od dvije velike menze. Svaki matemati£ar inzistira na tomeda ru£a u menzi u kojoj se nalazi paran broj njegovih prijatelja. Dokaºiteda je broj na£ina na koje se matemati£are moºe rasporediti u dvije menzepotencija broja 2.

14. (IMO SL 1998, C2) Pretpostavimo da imamo tablicu m×n realnih brojevatakvu da je suma svakog retka i suma svakog stupca cijeli broj. Dokaºite daje mogu¢e zaokuºiti na vi²e ili na manje (dakle, napraviti d·e ili b·c) svakikoeficijent tako da sume ostanu jednake.

15. (China West 2002)Neka su A1, A2, . . . , An+1 neprazni podskupovi od 1, 2, . . . , n. Dokaºiteda postoje neprazni i disjunktni I, J ⊆ 1, 2, . . . , n+ 1 takvi da je⋃

k∈I

Ak =⋃k∈J

Ak.

16. (Russia 2001) Na natjecanju s n zadataka je sudjelovalo m natjecatelja.Svaki zadatak je vrijedio odreen (prirodan) broj bodova i nisu se moglidobiti parcijalni bodovi. Nakon ²to su svi radovi bodovani, ispostavilo seda bi se mijenjanjem bodova na zadacima mogao posti¢i bilo koji (striktan)redoslijed natjecatelja. Koliko najvi²e moºe iznosti broj m, u odnosu na n?

17. (Russia 1998) Svako polje (2n − 1) × (2n − 1) plo£e sadrºi 1 ili −1. Takavraspored bojeva nazivamo uspje²nim ako je svaki broj jednak umno²kusvojih susjeda (polja koja s tim poljem imaju zajedni£ku stranicu). Odreditebroj uspje²nih rasporeda brojeva.

18. (Iran 2006) Neka je B ⊆ Zn3 sa svojstvom da za svaka dva razli£ita elementaiz B, a = (a1, a2, . . . , an) i b = (b1, b2, . . . , bn), postoji 1 ≤ i ≤ n takav daje ai = bi + 1 (u Z3). Dokaºite da je |B| ≤ 2n.

Page 35: Kako zapisati rje²enja? - Math · Kako zapisati rje²enja? IMO zadaci i bodovne sheme ... Nemojte zaboraviti trivijalne slu£ajeve (nulu, prazan skup, bazu indukcije) Pi²ite obja²njenja,

IMO pripreme 2014. & 2015. Vjekoslav Kova£, 25. 6. 2014. & 20. 6. 2015.

Vjerojatnosne i analiti£ke metode u kombinatorici

Ideja ovih zabilje²ki je okupiti osnovnu teoriju i primjere/zadatke iz kombinato-rike koji se tipi£no rje²avaju vjerojatnosnim i analiti£kim tehnikama. Materijalisu podijeljeni u tri cjeline, upravo prema naj£e²¢im trima takvim tehnikama:

1. vjerojatnosna metoda,

2. funkcije izvodnice,

3. Fourierova analiza.

Obzirom da su na pripremama godine 2014. bile obraene cjeline 2 i 3, a godine2015. cjelina 1, £inilo mi se prirodnim objediniti zabilje²ke iz tih dviju godina.

1. Vjerojatnosna metoda

Pod vjerojatnosnom metodom se obi£no podrazumijeva primjena vjerojatnosnihtehnika kod deterministi£kih problema, u £ijoj formulaciji nema ni£eg slu£ajnogpa takva primjena moºe biti donekle neo£ekivana. Malo preciznije, pretposta-vimo da ºelimo dokazati postojanje nekog dobrog objekta. Mi na skupu svihobjekata deniramo neku vjerojatnosnu mjeru pa potom pokaºemo da skup svihdobrih objekata ima pozitivnu mjeru, tj. da je pozitivna vjerojatnost da ¢e slu-£ajno odabrani objekt biti dobar. Kao posljedicu dobivamo da postoji baremjedan dobar objekt, ali ne mora odmah biti jasno kako ga doista na¢i. Korisnostvjerojatnosne metode najbolje se vidi iz brojnih primjera. Ponekad je potrebnoma²tovito preformulirati problem i denirati odgovaraju¢i vjerojatnosni prostor.Najve¢i promotor vjerojatnosne metode bio je Paul Erd®s.

Od vjerojatnosti ¢e nam trebati samo najosnovniji pojmovi. Prostor elementarnihdogaaja Ω ¢e kod nas uvijek biti neki kona£ni skup. Vjerojatnost je tada jed-nozna£no odreena nenegativnim brojevima (pω)ω∈Ω takvima da je

∑ω∈Ω pω = 1.

U tom slu£aju za svaki dogaaj A, ²to je naprosto proizvoljni podskup od Ω,deniramo njegovu vjerojatnost sa

P(A) :=∑ω∈A

pω ∈ [0, 1].

Po deniciji je o£iglednoP(Ac) = 1− P(A).

Lako je vidjeti da za bilo koje dogaaje A1, A2, . . . , An vrijedi

P( n⋃i=1

Ai

)≤

n∑i=1

P(Ai).

tovi²e, ako su A1, A2, . . . , An meusobno disjunktni, imamo £ak jednakost:

P( n⋃i=1

Ai

)=

n∑i=1

P(Ai).

Page 36: Kako zapisati rje²enja? - Math · Kako zapisati rje²enja? IMO zadaci i bodovne sheme ... Nemojte zaboraviti trivijalne slu£ajeve (nulu, prazan skup, bazu indukcije) Pi²ite obja²njenja,

Dogaaji A1, A2, . . . , An su meusobno nezavisni ako za svaki podskup I ⊆1, 2, . . . , n vrijedi

P(⋂i∈I

Ai

)=∏i∈I

P(Ai).

Intuitivno, pojavljivanje nekih od tih dogaaja ne utje£e na vjerojatnosti pojav-ljivanja ostalih dogaaja. U vjerojatnosti £esto umjesto P(A1 ∩ A2 ∩ · · · ∩ An)pi²emo naprosto P(A1, A2, . . . , An). Na Ω se naj£e²¢e pretpostavlja tzv. unifor-mna vjerojatnost, tj. da je pω = 1/|Ω| za svaki ω ∈ Ω, ²to zna£i da za svakidogaaj A vrijedi

P(A) =|A||Ω|

=broj povoljnih mogu¢nosti

broj svih mogu¢nosti.

Mi ¢emo podrazumijevati da je to slu£aj kad god nije re£eno druga£ije.

Slu£ajna varijabla je ovdje naprosto proizvoljna funkcija X : Ω → R. Njenoo£ekivanje je dano formulom

EX :=∑ω∈Ω

pωX(ω).

Ukoliko X poprima meusobno razli£ite vrijednosti a1, . . . , am ∈ R, tada je

EX =m∑i=1

P(X = ai)ai

te op¢enitije

Ef(X) =m∑i=1

P(X = ai)f(ai)

za bilo koju funkciju f : R → R. Kaºemo da brojevi P(X = a), a ∈ R odreujurazdiobu od X. O£ekivanje je linearno, tj.

E(αX + βY ) = αEX + βEY

za slu£ajne varijable X, Y i brojeve α, β ∈ R. Osim toga je i monotono, tj. akoza slu£ajne varijable X i Y vrijedi X ≤ Y , tada je EX ≤ EY . Trivijalna (alinama korisna) opservacija je da ako za neki a ∈ R vrijedi EX ≥ a (respektivnoEX ≤ a), tada postoji barem jedan ω ∈ Ω takav da je X(ω) ≥ a (respektivnoX(ω) ≤ a). Slu£ajne varijable X1, X2, . . . , Xn su meusobno nezavisne ako zasvaki izbor a1, a2, . . . , an ∈ R vrijedi

P(X1 = a1, X2 = a2, . . . , Xn = an) =n∏i=1

P(Xi = ai).

U tom slu£aju imamo

E( n∏i=1

Xi

)=

n∏i=1

EXi

te, op¢enitije, za proizvoljne funkcije f1, f2, . . . , fn : R→ R vrijedi

E( n∏i=1

fi(Xi))

=n∏i=1

Efi(Xi).

Poznata (i prili£no jednostavna) £injenica je da uvijek moºemo konstruirati pros-tor Ω i vjerojatnost P tako da X1, X2, . . . , Xn budu meusobno nezavisne slu£ajnevarijable s unaprijed zadanim razdiobama.

Page 37: Kako zapisati rje²enja? - Math · Kako zapisati rje²enja? IMO zadaci i bodovne sheme ... Nemojte zaboraviti trivijalne slu£ajeve (nulu, prazan skup, bazu indukcije) Pi²ite obja²njenja,

Zadaci

1. (Kraftova nejednakost) Neka je F kona£na kolekcija binarnih stringova ko-na£ne duljine takva da nikoji string iz F nije preks (tj. po£etni dio) nekogdrugog stringa iz F . Ako je Ni broj stringova u F duljine i, dokaºite∑

iNi2i≤ 1.

Rje²enje. Neka je n najve¢a duljina nekog stringa iz F . Za prostor ele-mentarnih dogaaja Ω uzimamo skup svih stringova s duljine n. Za svakif ∈ F promatramo dogaaj

Af := s ∈ Ω : string f je preks stringa s.

Primijetimo da zbog slu£ajnosti of s imamo

P(Af ) =broj stringova duljine n s unaprijed odreenih |f | bitova

broj stringova duljine n

=2n−|f |

2n=

1

2|f |,

pri £emu nam |f | ozna£ava duljinu stringa f .

Dogaaji Af i Ag su meusobno disjunktni za razli£ite f, g ∈ F . Naime, kadbi postojao string s ∈ Af∩Ag i ako bez smanjenja op¢enitosti pretpostavimo|f | ≤ |g|, tada bi f i g istovremeno bili preksi od s, ²to bi zna£ilo da jef preks od g, a to je protivno pretpostavci na F . Zbog disjunktnostidogaaja Af moºemo ra£unati:

1 ≥ P( ⋃f∈F

Af

)=∑f∈F

P(Af ) =∑f∈F

1

2|f |=∑i≥0

Ni

2i.

2. (Spernerov teorem) Neka je F familija podskupova od 1, 2, . . . , n takvada ne postoje A,B ∈ F za koje bi vrijedilo A $ B. Dokaºite |F| ≤

(nbn/2c

).

Rje²enje, prilagoeno iz knjige [1], poglavlje 11. Neka je Ω skup svih per-mutacija σ skupa 1, 2, . . . , n. Za svaki A ∈ F deniramo dogaaj

CA := σ ∈ Ω : svaki element od A prethodi svakom elementuod Ac u slijedu σ(1), σ(2), . . . , σ(n).

Drugim rije£ima, ako je |A| = k, tada σ ∈ CA zna£i

A = σ(1), σ(2), . . . , σ(k).

Za svaki A ∈ F zbog slu£ajnosti od σ imamo:

P(CA) =(broj permutacija od A)(broj permutacija od Ac)

broj permutacija od 1, 2, . . . , n

=|A|!(n− |A|)!

n!=

1(n|A|

) ≥ 1(nbn/2c

) ,radi poznatog svojstva binomnih koecijenata: max0≤k≤n

(nk

)=(

nbn/2c

), tj.

najve¢i element svakog retka Pascalovog trokuta nalazi se u sredini.

Page 38: Kako zapisati rje²enja? - Math · Kako zapisati rje²enja? IMO zadaci i bodovne sheme ... Nemojte zaboraviti trivijalne slu£ajeve (nulu, prazan skup, bazu indukcije) Pi²ite obja²njenja,

S druge strane, za svake A,B ∈ F , A 6= B dogaaji CA i CB morajubiti disjunktni. Naime, pretpostavimo da postoji ω ∈ CA ∩ CB i ozna£imok = |A|, l = |B|. Neka je bez smanjenja op¢enitosti k ≤ l. To zna£i daje A = σ(1), σ(2), . . . , σ(k) i B = σ(1), σ(2), . . . , σ(k), . . . , σ(l), iz £egaslijedi A ⊆ B, ²to je kontradikcija. Kona£no, iz disjunktnosti dogaaja CAslijedi

1 ≥ P( ⋃A∈F

CA

)=∑A∈F

P(CA) ≥ |F|(nbn/2c

) ,²to nam daje |F| ≤

(nbn/2c

).

Primijetimo da je ograda(

nbn/2c

)optimalna, jer se postiºe ako je F familija

svih podskupova od 1, 2, . . . , n s to£no bn/2c elemenata.

3. (Zadatak iz £lanka [2]) U svaku ku¢icu 100× 100 tablice upisan je jedan odbrojeva 1, 2, 3, . . . , 5000 i to tako da se svaki od tih brojeva pojavljuje to£nodvaput. Dokaºite da je mogu¢e odabrati 100 ku¢ica tablice tako da vrijedi:

• u svakom retku je odabrana to£no jedna ku¢ica,

• u svakom stupcu je odabrana to£no jedna ku¢ica,

• brojevi u odabranim ku¢icama su meusobno razli£iti.

Rje²enje. Uzmimo slu£ajnu permutaciju a1, a2, . . . , a100 od 1, 2, . . . , 100,tj. Ω je kolekcija svih permutacija skupa 1, 2, . . . , 100. Pretpostavljamoda su sve permutacije jednako vjerojatne, tj. pojavljuju se s vjerojatno²¢u1/100!. Odaberemo li ku¢ice (j, aj); j = 1, 2, . . . , 100, one ¢e o£iglednozadovoljavati prva dva uvjeta. Neka je Ak dogaaj da smo odabrali objeku¢ice s brojem k za ksirani k ∈ 1, 2, . . . , 5000. Ako se te ku¢ice nalazeu razli£itim recima i razli£itim stupcima, onda je P(Ak) = 1

100· 1

99, dok je

ina£e P(Ak) = 0. Dakle,

P( 5000⋃k=1

Ak

)≤

5000∑k=1

P(Ak) ≤ 5000 · 1

100· 1

99=

50

99

pa je

P( 5000⋂k=1

Ack

)≥ 49

99> 0.

Slijedi da za barem jedan odabir permutacije nemamo ponavljanja brojevau odgovaraju¢im ku¢icama.

4. (IMO Shortlist 2006. C3 ) Neka je S kona£ni skup to£aka u ravnini takavda nikoje tri od njih nisu kolinearne. Za svaki konveksni mnogokut M svrhovima iz S neka a(M) ozna£ava broj njegovih vrhova, a b(M) broj to£akaiz S koje leºe izvan mnogokuta M . Napomenimo da se prazan skup, to£kai duºina shva¢aju kao konveksni mnogokuti s redom 0, 1 i 2 vrha. Dokaºiteda za svaki x ∈ R vrijedi∑

M je konveksni mnogokutvrhovi od M su iz S

xa(M)(1− x)b(M) = 1.

Page 39: Kako zapisati rje²enja? - Math · Kako zapisati rje²enja? IMO zadaci i bodovne sheme ... Nemojte zaboraviti trivijalne slu£ajeve (nulu, prazan skup, bazu indukcije) Pi²ite obja²njenja,

Rje²enje iz [3]. Primijetimo da je lijeva strana polinom u varijabli x. Po te-oremu o jednakosti polinoma, dovoljno je jednakost provjeriti za beskona£nomnogo razli£itih vrijednosti od x, a mi ¢emo to u£initi za sve x ∈ [0, 1].

Za Ω uzmimo skup svih bojenja to£aka iz S u dvije boje (crvenu i plavu),pri £emu to£ke bojimo nezavisno, a svaka to£ka biva obojena crveno s vjero-jatnosti x, odnosno plavo s vjerojatnosti 1−x. Za svaki mnogokut M nekaAM ozna£ava dogaaj da su mu svi vrhovi obojeni crveno, a sve to£ke izvanM su obojene plavo. Najprije primijetimo da su svi ti dogaaji meusobnodisjunktni i u uniji daju cijeli Ω. Drugim rije£ima, svako bojenje se nalaziu to£no jednom od dogaaja AM . Naime, za svako bojenje odgovaraju¢imnogokut je naprosto konveksna ljuska svih crvenih to£aka iz S. Primi-jetimo da za svaki mnogokut M vrijedi P(AM) = xa(M)(1 − x)b(M) pa jedoista ∑

M

xa(M)(1− x)b(M) =∑M

P(AM) = P(Ω) = 1.

5. (Zadatak prilagoen iz knjige [1]) Neka su v1, v2, . . . , vn ∈ Rn takvi da je‖vi‖ ≤ 1, i = 1, 2, . . . , n, pri £emu ‖ · ‖ ozna£ava euklidsku normu. Dokaºiteda za svake brojeve p1, p2, . . . , pn ∈ [0, 1] postoje ε1, ε2, . . . , εn ∈ 0, 1 takvida vrijedi ∥∥∥ n∑

i=1

εivi −n∑i=1

pivi

∥∥∥ ≤ √n2.

Rje²enje. Fiksirajmo koecijente p1, . . . , pn. Neka su ε1, . . . , εn nezavisneslu£ajne varijable koje poprimaju vrijednosti u 0, 1 i £ije razdiobe su danesa

P(εi = 0) = 1− pi, P(εi = 1) = pi

za i = 1, . . . , n. Promatramo slu£ajnu varijablu

X :=∥∥∥ n∑i=1

εivi −n∑i=1

pivi

∥∥∥2

=n∑

i,j=1

(εi − pi)(εj − pj)〈vi, vj〉,

pri £emu 〈·, ·〉 ozna£ava standardni skalarni produkt. Primijetimo da je

E(εi − pi) = (1− pi)(0− pi) + pi(1− pi) = 0

teE(εi − pi)2 = (1− pi)(0− pi)2 + pi(1− pi)2 = pi(1− pi).

Nadalje, zbog nezavisnosti za i 6= j imamo

E(εi − pi)(εj − pj) = E(εi − pi)E(εj − pj) = 0.

Zato je

EX =n∑i=1

‖vi‖2E(εi − pi)2 ≤n∑i=1

pi(1− pi)︸ ︷︷ ︸≤1/4

≤ n

4.

Prema tome, postoji ω ∈ Ω takav da za brojeve

εi := εi(ω) ∈ 0, 1

Page 40: Kako zapisati rje²enja? - Math · Kako zapisati rje²enja? IMO zadaci i bodovne sheme ... Nemojte zaboraviti trivijalne slu£ajeve (nulu, prazan skup, bazu indukcije) Pi²ite obja²njenja,

vrijedi ∥∥∥ n∑i=1

εivi −n∑i=1

pivi

∥∥∥2

= X(ω) ≤ n

4,

²to je trebalo dokazati.

Primijetimo da je ograda√n/2 optimalna za svaki prirodni broj n, jer

se postiºe kada su vi vektori standardne ortonormirane baze od Rn, a svikoecijenti su pi = 1/2. Zapravo, u tom slu£aju nam svi izbori od εi dajuistu vrijednost

√n/2.

6. (IMO Shortlist 1999. C4 ) Neka je A neki skup od n ostataka modulo n2.Pokaºite da postoji skup B od n ostataka modulo n2 takav da je barempola ostataka modulo n2 oblika a + b za a ∈ A i b ∈ B. (Napomenimo dase ostaci i zbrajaju modulo n2.)

Rje²enje iz [3]. Nezavisno i uniformno biramo n ostataka modulo n2, tj.preciznije, Ω je skup svih n-torki ostataka modulo n2. Ako je B skup kojise sastoji od odabranih ostataka (koji se mogu i ponavljati), tada B imanajvi²e n elemenata, a ima smisla promatrati i slu£ajnu varijablu X kojaje jednaka broju svih ostataka prikazivih u obliku a + b za a ∈ A i b ∈ B,tj. X := card(A+B).

Za svaki ostatak i postoji to£no n ostataka j takvih da je i ∈ A + j ⇔j ∈ i− A. Zato je

P(i 6∈ A+B) =(n2 − n

n2

)n=(

1− 1

n

)npa imamo

P(i ∈ A+B) = 1−(

1− 1

n

)n.

Dakle,

EX =∑i

E1i∈A+B = n2(

1−(

1− 1

n

)n)≥ n2/2.

Naime,(1 − 1

n

)n ≤ 12je ekvivalentno s 2−1/n ≥ 1 − 1

n, ²to pak slijedi iz

poznate ocjene ex ≥ 1 +x za x ∈ R. Prema tome, mogu¢e je odabrati n (ilimanje) ostataka i okupiti ih u skup B s traºenim svojstvom.

7. (Iran, selekcijski test 2008. #6 ) Na nekom turniru sudjeluje 799 natjecate-lja i poznato je da svaka dva natjecatelja igraju to£no jedan me£ u kojempobjeuje jedan od njih. Dokaºite da postoje skupovi A i B od po 7 na-tjecatelja takvi da je svaki natjecatelj iz A pobijedio svakog natjecatelja izB.

Rje²enje iz [3]. Za svakog natjecatelja j ∈ 1, 2, . . . , 799 ozna£imo s njbroj natjecatelja koji su ga pobijedili. Neka je Ω kolekcija svih sedmo£la-nih skupova natjecatelja. Neka je X slu£ajna varijabla koja broji kolikonatjecatelja su pobijedili svi £lanovi slu£ajno odabranog skupa, tj. za svakiA ∈ Ω deniramo

X(A) := cardj : svaki £lan od A je pobijedio j.

Page 41: Kako zapisati rje²enja? - Math · Kako zapisati rje²enja? IMO zadaci i bodovne sheme ... Nemojte zaboraviti trivijalne slu£ajeve (nulu, prazan skup, bazu indukcije) Pi²ite obja²njenja,

Osim toga, za svakog natjecatelja j moºemo denirati slu£ajnu varijablu

Yj(A) :=

1 ako su svi £lanovi od A pobijedili j,0 ina£e.

O£igledno je X =∑799

j=1 Yj te P(Yj = 1) =(nj7

)/(

7997

)pa je

EYj =

(nj7

)/(799

7

)i EX =

799∑j=1

(nj7

)/(799

7

).

Kako znamo da je

799∑j=1

nj = broj me£eva =

(799

2

), tj.

1

799

799∑j=1

nj = 399,

zbog konveksnosti funkcije N0 → R, x 7→(x7

)imamo

1

799

799∑j=1

(nj7

)≥(

399

7

).

(Za primjenu Jensenove nejednakosti bismo zapravo trebali imati konveksnufunkciju na [0,+∞〉, no to se lako postigne tako da se funkcija na N0 linearnointerpolira izmeu susjednih cijelih brojeva.) Zato je

EX ≥ 799 ·(

399

7

)/(799

7

)= 6.02549 . . . ,

odakle zbog cjelobrojnosti slijedi da postoji sedmo£lani skup A takav da jeX(A) ≥ 7 te za skup B sada naprosto treba uzeti nekih 7 natjecatelja kojesu pobijedili svi natjecatelji iz A.

8. (Erd®s) Ramseyev broj R(k, k) je najmanji prirodni broj n takav da zasvako bojenje bridova od Kn (tj. potpunog grafa s n vrhova) u dvije boje,crvenu i plavu, postoji k vrhova koji su povezani samo crvenom bojom ilipostoji k vrhova koji su povezani samo plavom bojom. Mogu¢e je pokazatida takav broj R(k, k) uvijek postoji, ali moºe biti vrlo velik. Kao primjere jelako vidjeti R(2, 2) = 2 (trivijalno) i R(3, 3) = 6 (£esti zada£i¢ za osnovnu²kolu). Dokaºite da za svaki k ∈ N, k ≥ 4 vrijedi R(k, k) ≥ 2k/2.

Rje²enje, prilagoeno iz knjige [1], poglavlje 1. Stavimo n = b2k/2c. Nekaje Ω skup svih bojenja bridova od Kn u crvenu ili plavu boju. Dakle,|Ω| = 2(n2). Za ksirani podskup S skupa vrhova ozna£imo sa AS dogaajda su svi bridovi s krajevima u S istobojni. Ako je |S| = k, tada imamo

P(AS) =2 · 2(n2)−(k2)

2(n2)= 21−(k2).

Zato je vjerojatnost da se dogodio barem jedan od dogaaja AS za nekik-£lani skup vrhova S mogu¢e ocijeniti:

P( ⋃

S|S|=k

AS

)≤∑S|S|=k

P(AS) =

(n

k

)21−(k2).

Page 42: Kako zapisati rje²enja? - Math · Kako zapisati rje²enja? IMO zadaci i bodovne sheme ... Nemojte zaboraviti trivijalne slu£ajeve (nulu, prazan skup, bazu indukcije) Pi²ite obja²njenja,

Ukoliko jo² pokaºemo(nk

)21−(k2) < 1, tada ¢e biti

P( ⋂

S|S|=k

AcS

)= P

(( ⋃S|S|=k

AS

)c)> 0

pa ¢e postojati barem jedno bojenje za koje niti jedan podgraf odreenskupom vrhova S takvim da je |S| = k nema istobojne bridove. To ¢ezna£iti R(k, k) > n, £ime ¢e biti dokazana traºena tvrdnja R(k, k) ≥ 2k/2.

Ocijenimo: (n

k

)21−(k2) =

n(n− 1) · · · (n− k + 1)

k!21− k(k−1)

2

<nk

k!2−

k2

2+ k

2+1 =

nk

2k2

2

2k2

+1

k!< 1.

Pritom smo koristili

nk = b2k/2ck ≤ (2k/2)k = 2k2/2

te nejednakostk! ≥ 2k/2+1

za k ∈ N, k ≥ 4, koju je lako dokazati indukcijom.

9. (Zadatak za vjeºbu iz knjige [1]) Neka je (V,E) bipartitni graf s n vrhova ineka je svakom vrhu v ∈ V pridruºena kona£na lista S(v) od |S(v)| > log2 ndozvoljenih boja. Dokaºite da je mogu¢e obojiti vrhove grafa (svaki u nekuod njemu dozvoljenih boja) tako da nikoja dva istobojna vrha nisu spojenabridom.

Rje²enje. Neka je k najmanji prirodni broj strogo ve¢i od log2 n, tj. k =blog2 nc+1, tako da imamo |S(v)| ≥ k za svaki v ∈ V . Da je graf bipartitanzna£i da mu se skup vrhova V moºe particionirati u dva skupa V1 i V2

takva da svaki brid iz E ima jedan kraj u V1, a drugi u V2. Ozna£imo saS :=

⋃v∈V S(v) skup svih boja koje se pojavljuju u listama.

Neka je Ω = P(S), tj. sastoji se od svih podskupova skupa boja. Pretpos-tavljamo uniformnu vjerojatnost na Ω, ²to ima za posljedicu da se svakaboja c ∈ S pojavljuje s vjerojatnosti 1/2 u slu£ajno odabranom skupuT ∈ P(S) te s vjerojatnosti 1/2 u njegovom komplementu T c. Osim togasu dogaaji T ∈ Ω : c ∈ T meusobno nezavisni za razli£ite c ∈ S. Naszapravo zanimaju dogaaji

Av := T ∈ Ω : T ∩ S(v) = ∅ za v ∈ V1,

Av := T ∈ Ω : T c ∩ S(v) = ∅ za v ∈ V2.

Primijetimo da za svaki v ∈ V (neovisno je li u V1 ili V2) vrijedi

P(Av) =broj podskupova od S \ S(v)

broj podskupova od S=

2|S\S(v)|

2|S|= 2−|S(v)| ≤ 2−k <

1

n.

Dakle,

P( ⋃v∈V

Av

)≤∑v∈V

P(Av) < n · 1

n= 1

Page 43: Kako zapisati rje²enja? - Math · Kako zapisati rje²enja? IMO zadaci i bodovne sheme ... Nemojte zaboraviti trivijalne slu£ajeve (nulu, prazan skup, bazu indukcije) Pi²ite obja²njenja,

pa jeP( ⋂v∈V

Acv

)> 0.

Slijedi da postoji skup T ∈ Ω koji se ne nalazi niti u jednom od dogaajaAv, v ∈ V . Za svaki vrh v ∈ V1 imamo T ∩S(v) 6= ∅ pa v moºemo obojiti uneku od boja iz T . S druge strane, za svaki v ∈ V2 imamo T c∩S(v) 6= ∅ pav moºemo obojiti u neku od boja iz T c. Dobiveno bojenje ne moºe kreiratibridove s istobojnim krajevima, naprosto jer su u V1 kori²tene samo boje izT , a u V2 samo boje iz T c.

Zadaci za samostalan rad

1. (Zadatak iz knjige [1], poglavlje 15 ) Neka je n ≥ 4 prirodan broj. Dokaºiteda postoji bojenje bridova potpunog grafa Kn u dvije boje koje daje najvi²e(n4

)/32 jednobojnih kopija grafa K4, tj. takvih podgrafova s £etiri vrha da

su svih 6 njihovih bridova iste boje.

Rje²enje. Za Ω uzmimo sva bojenja bridova od Kn u crvenu i plavu. Uslu£aju uniformne vjerojatnosti su obje boje jednako vjerojatne za svakivrh te su bojenja vrhova meusobno nezavisna. Za svaki skup S od 4 vrhaneka je AS dogaaj da je pripadni podgraf (induciran sa S) jednobojan,tj. da su svi bridovi s krajevima iz S iste boje. O£igledno je P(AS) =2 · (1

2)6 = 2−5. Nadalje, neka je N slu£ajna varijabla koja broji jednobojne

kopije od K4, tj. N :=∑|S|=4 1AS . Kako imamo EN =

∑|S|=4 P(AS) =(

n4

)· 2−5, zaklju£ujemo da postoji bojenje ω ∈ Ω koje daje najvi²e

(n4

)· 2−5

jednobojnih kopija od K4.

2. (Zadatak za vjeºbu iz knjige [1]) Neka je (Ai, Bi) : i = 1, 2, . . . ,m familijaparova skupova cijelih brojeva takva da za svaki i vrijedi |Ai| = k, |Bi| = `,Ai ∩Bi = ∅ te da za svake i 6= j vrijedi (Ai ∩Bj)∪ (Aj ∩Bi) 6= ∅. Dokaºitem ≤ (k+`)k+`

kk``.

Rje²enje. Stavimo S :=⋃mi=1(Ai ∪ Bi). Za Ω uzmimo sve funkcije f : S →

1, 2, . . . , k + ` i pretpostavimo uniformnu vjerojatnost. Promotrimo do-gaaje

Ci :=f ∈ Ω : f(Ai) ⊆ 1, . . . , k, f(Bi) ⊆ k + 1, . . . , k + `

za i = 1, 2, . . . ,m. Obzirom da su Ai i Bi disjunktni, imamo

P(Ci) =(br. fja sa Ai u 1, . . . , k)(br. fja sa Bi u k + 1, . . . , k + `)

broj funkcija sa Ai ∪Bi u 1, . . . , k + `

=kk``

(k + `)k+`.

Naime, broj odabira vrijednosti od f na S \ (Ai ∪ Bi) se pokrati kod pre-brajanja. Tvrdimo da su ti dogaaji disjunktni. Naime, kad bi posto-jala f ∈ Ci ∩ Cj za neke i 6= j, tada bismo imali f(Ai) ∩ f(Bj) = ∅ if(Aj) ∩ f(Bi) = ∅, ²to povla£i Ai ∩ Bj = ∅ i Aj ∩ Bi = ∅, a to je u kon-tradikciji s pretpostavkom zadatka. Kona£no, traºena nejednakost slijedi

Page 44: Kako zapisati rje²enja? - Math · Kako zapisati rje²enja? IMO zadaci i bodovne sheme ... Nemojte zaboraviti trivijalne slu£ajeve (nulu, prazan skup, bazu indukcije) Pi²ite obja²njenja,

iz

1 ≥ P( m⋃i=1

Ci

)=

m∑i=1

P(Ci) = m · kk``

(k + `)k+`.

3. (Erd®s) Hipergraf je ureeni par (V,E) kona£nog skupa vrhova V i familijeE podskupova od V koje zovemo bridovi. Ako je (V,E) hipergraf u kojemsvaki brid ima to£no n ≥ 2 vrhova i bridova ima |E| ≤ 2n−1, dokaºite da setada vrhovi V mogu obojiti u dvije boje tako da svaki brid sadrºi vrhoveobiju boja.

Rje²enje iz [4]. Neka Ω £ine sva bojenja vrhova V u crvenu ili plavu boju.Uz pretpostavku uniforme vjerojatnosti je svaki vrh v ∈ V obojen u crvenuili plavu s vjerojatnostima 1/2, a boje razli£itih vrhova su nezavisne. Nekaslu£ajna varijabla N ozna£ava broj jednobojnih bridova iz E obzirom napromatrano slu£ajno bojenje.

EN =∑e∈E

E1e je istobojan

=∑e∈E

P(e je istobojan) =∑e∈E

2 ·(1

2

)n=|E|2n−1

≤ 1

Dakle, EN ≤ 1. Razlikujemo dva slu£aja.

Slu£aj 1. Ako je P (N = 0) = 0, tada je zapravo N ≥ 1. Ako bi za nekiω ∈ Ω bilo N(ω) > 1, tada bismo imali EN > 1, ²to bi bilo u kontradikciji snetom dobivenim EN ≤ 1. Dakle, N je identi£ki jednaka 1, tj. svako bojenjedaje to£no jedan jednobojni brid. Ipak, to nije mogu¢e. Naime, ako postojisamo jedan brid u E, tada je neka dva njegova vrha mogu¢e razli£ito obojitipomo¢u nekog ω pa bi bilo N(ω) = 0. Ako pak E sadrºi barem dva brida,tada bi bojenje ω svih vrhova u crvenu dalo N(ω) = |E| ≥ 2. Kontradikcija.

Slu£aj 2. Neka je dakle P (N = 0) > 0. To zna£i da postoji bojenje ω takvoda je N(ω) = 0, tj. bojenje koje ne daje niti jedan jednobojni brid.

4. (Zadatak za vjeºbu iz knjige [1]) Ako je (V,E) hipergraf takav da je |V | = n,|E| = m ≥ n

3i svaki brid sadrºi to£no 3 vrha, dokaºite da postoji skup

S ⊆ V takav da je |S| ≥ 2n3/2

3√

3mi da niti jedan brid iz E nije podskup skupa

S.

Rje²enje. Neka prostor Ω £ine svi podskupovi skupa vrhova, tj. Ω = P(V ).Ovog puta konstruiramo vjerojatnost tako da za svaki vrh v slu£ajni skupS sadrºi v s vjerojatnosti p, tj. P(S ∈ Ω : v ∈ S) = p, te da su svidogaaji S ∈ Ω : v ∈ S, v ∈ V meusobno nezavisni. (Dakle, bacanjemnesimetri£nog nov£i¢a za svaki v odlu£ujemo ho¢emo li ga staviti u S ili uSc.) Parametar p ∈ [0, 1] ¢e biti odabran kasnije.

Promotrimo slu£ajne varijable:

X := |S| = broj odabranih vrhova,Y := |E ∩ P(S)| = broj bridova koji su podskupovi od S,Xv := 1S∈Ω:v∈S za v ∈ V,Ye := 1S∈Ω:e⊆S za e ∈ E,

Page 45: Kako zapisati rje²enja? - Math · Kako zapisati rje²enja? IMO zadaci i bodovne sheme ... Nemojte zaboraviti trivijalne slu£ajeve (nulu, prazan skup, bazu indukcije) Pi²ite obja²njenja,

tako da imamo X =∑

v∈V Xv, Y =∑

e∈E Ye. Kako je EXv = P(S ∈Ω : v ∈ S) = p i EYe = P(S ∈ Ω : e ⊆ S) = p|e| = p3, linearnosto£ekivanja daje E(X−Y ) = np−mp3. Dakle, postoji skup S ∈ Ω takav daje X(S)− Y (S) ≥ np−mp3. Za svaki brid iz E ∩ P(S) moºemo odabratijedan njegov vrh te ga izbaciti iz skupa S. Skup koji preostane imat ¢ebarem np −mp3 vrhova i ne¢e sadrºavati (kao nadskup) niti jedan brid izE.

Preostaje odabrati p tako da maksimizira funkciju f : [0, 1] → R, f(p) :=np − mp3. Ra£unanjem derivacije f ′(p) = n − 3mp2 lako nalazimo da senjen maksimum postiºe u p =

√n

3m, kada je f(p) = np−mp3 = 2n3/2

3√

3m.

5. (Erd®s) Za svaki kona£ni skup B ⊆ Z \ 0 postoji podskup A ⊆ B takavda je |A| ≥ |B|

3i da ne postoje x, y, z ∈ A za koje bi vrijedilo x+ y = z.

Rje²enje iz knjige [1], poglavlje 1. Fiksirajmo neki prosti broj p = 3k + 2koji je ve¢i od 2 maxx∈B |x|. Promatrajmo skup B modulo p, tj. smatrajmoga podskupom kona£nog polja Z/pZ. Dovoljno je na¢i podskup A od Btakav da je |A| ≥ |B|/3 i da jednadºba x + y = z (u Z/pZ, tj. modulo p)nema rje²enja u skupu A.

Uzmimo Ω = (Z/pZ) \ 0 s uniformnom vjerojatnosti i neka je N slu£ajnavarijabla denirana sa

N(a) := broj elemenata od aB ∩ k + 1, . . . , 2k + 1

za svaki a ∈ Ω. Imamo

EN =∑b∈B

E1a∈Ω : ab∈k+1,...,2k+1 =∑b∈B

E1a∈Ω : a∈(k+1)b−1,...,(2k+1)b−1

=∑b∈B

k + 1

3k + 1= |B| · k + 1

3k + 1≥ |B|

3

pa postoji a ∈ Z/pZ, a 6= 0 takav da je N(a) ≥ |B|/3, tj. da skup aB ∩k + 1, . . . , 2k + 1 ima barem |B|/3 elemenata. Tada stavimo

A := B ∩ a−1(k + 1), . . . , a−1(2k + 1).

Kad bi postojali x, y, z ∈ A takvi da je x+y = z, tada bi vrijedilo ax+ay =az, ali je i ax, ay, az ∈ k + 1, . . . , 2k + 1, a zbroj modulo p nikoja dvabroja iz k + 1, . . . , 2k + 1 ne moºe upasti u taj isti skup. Dakle, A imasva traºena svojstva.

6. (Zadatak iz knjige [1], poglavlje 15 ) Neka je [ai,j]1≤i,j≤n matrica tipa n× ns elementima iz [−1, 1]. Dokaºite da postoje ε1, . . . , εn ∈ −1, 1 takvi davrijedi

∣∣∑nj=1 εjai,j

∣∣ ≤√2n ln(2n) za sve i = 1, . . . , n.

Rje²enje. Neka su ε1, . . . , εn nezavisne slu£ajne varijable s vrijednostimau −1, 1 koje imaju simetri£nu razdiobu, tj. P(εi = −1) = P(εi = 1) =12. Promatramo slu£ajne varijable Xi :=

∑nj=1 εjai,j za i = 1, 2, . . . , n.

Page 46: Kako zapisati rje²enja? - Math · Kako zapisati rje²enja? IMO zadaci i bodovne sheme ... Nemojte zaboraviti trivijalne slu£ajeve (nulu, prazan skup, bazu indukcije) Pi²ite obja²njenja,

Ozna£imo β :=√

2n ln(2n) i α := β/n. Zbog nezavisnosti moºemo ra£unati

EeαXi = Eeα∑nj=1 εjai,j = E

n∏j=1

eεjαai,j =n∏j=1

Eeεjαai,j

=n∏j=1

(12· eαai,j + 1

2· e−αai,j

)=

n∏j=1

ch(αai,j)

≤n∏j=1

eα2a2i,j/2 ≤

n∏j=1

eα2/2 = e

α2n2 = e

β2

2n

(zapravo uz strogu nejednakost na barem jednom od gornja dva mjesta) tepotom ocjenjujemo vjerojatnosti dogaaja ω ∈ Ω : |Xi(ω)| > β kao

P(|Xi| > β) = 2P(Xi > β) = 2P(eαXi > eαβ)

≤ 2e−αβEeαXi < 2e−β2

n eβ2

2n = 2e−β2

2n .

Ovdje smo koristili poznatu Markov-ebi²evljevu nejednakost koja kaºe daza slu£ajnu varijablu Y i za γ > 0 vrijedi P(|Y | > γ) ≤ γ−1E|Y |. Dakle,

P( n⋃i=1

|Xi| > β)≤

n∑i=1

P(|Xi| > β) < 2ne−β2

2n = 2ne− ln(2n) = 1.

Stoga imamo P(⋂n

i=1|Xi| ≤ β)> 0 pa postoji ω ∈ Ω takav da je

|Xi(ω)| ≤ β za i = 1, . . . , n, ²to zna£i da brojevi εi := ε(ω) zadovoljavajunejednakosti iz zadatka.

Kori²tena literatura i dodatni izvori zadataka

[1] N. Alon, J. H. Spencer, The Probabilistic Method, Wiley-Interscience Seriesin Discrete Mathematics and Optimization, tre¢e izdanje, John Wiley &Sons, Inc., 2008.

[2] L. K. Ho, Probabilistic Method, Mathematical Excalibur, 14, no.3, 2009.

[3] P.-S. Loh, Probabilistic Methods in Combinatorics, 2009.

[4] A. Milo²evi¢, Primjene vjerojatnosne metode na deterministi£ke igre, di-plomski rad, PMFMatemati£ki odsjek, Sveu£ili²te u Zagrebu, 2015.

Page 47: Kako zapisati rje²enja? - Math · Kako zapisati rje²enja? IMO zadaci i bodovne sheme ... Nemojte zaboraviti trivijalne slu£ajeve (nulu, prazan skup, bazu indukcije) Pi²ite obja²njenja,

2. Funkcije izvodnice

Funkcije izvodnice su korisno analiti£ko orue za prou£avanje nizova, posebnoonih koji dolaze iz raznih kombinatornih problema prebrojavanja. Kao ²to ¢emovidjeti, usko su povezane uz rekurzivne relacije, traºenje zatvorenih formula teispitivanje asimptotskog pona²anja.

Obi£na funkcija izvodnica (ili samo funkcija izvodnica) niza (an)∞n=0 je red poten-cija

A(x) :=∞∑n=0

anxn.

Eksponencijalna funkcija izvodnica niza (an)∞n=0 je obi£na funkcija izvodnica od(ann!

)∞n=0

, tj.

E(x) :=∞∑n=0

anxn

n!.

Ako niz (an)∞n=0 raste razumno brzo, tada su funkcije A i E dobro denirane nanekom intervalu 〈−r, r〉 makar za dovoljno mali r > 0 i ponekad se mogu izra£u-nati eksplicitne formule za A(x) i E(x). Razumno brzo u slu£aju obi£ne funkcijeizvodnice zna£i: ne brºe od svakog geometrijskog niza. Posebno je fascinantno ²tou nekim slu£ajevima imamo vrlo lijepe formule za A(x) ili E(x) makar je te²ko(ili £ak nemogu¢e) napisati zatvorenu formulu za an. Obratno, iz formule za A(x)ili E(x) se razvojem u red potencija (barem na£elno) mogu i²£itati koecijenti tedobiti formula za an. Ako je A(x) =

∑∞n=0 anx

n, tada obratno pi²emo

an = [xn]A(x),

tj. [xn]A(x) nam ozna£ava koecijent uz xn u razvoju od A(x).

Slijede neki poznati razvoji koji se mogu koristiti.

ex = 1 + x+x2

2!+x3

3!+ . . . =

∞∑n=0

xn

n!, za x ∈ R

cosx = 1− x2

2!+x4

4!− x6

6!+ . . . =

∞∑n=0

(−1)nx2n

(2n)!, za x ∈ R

sinx = x− x3

3!+x5

5!− x7

7!+ . . . =

∞∑n=0

(−1)nx2n+1

(2n+ 1)!, za x ∈ R

1

1− x= 1 + x+ x2 + x3 + . . . =

∞∑n=0

xn, za |x| < 1

1

(1− x)2= 1 + 2x+ 3x2 + 4x3 + . . . =

∞∑n=0

(n+ 1)xn, za |x| < 1

1

(1− x)k+1=∞∑n=0

(n+ k

k

)xn, za |x| < 1, k ∈ N0

(1 + x)α = 1 +

1

)x+

2

)x2 + . . . =

∞∑n=0

n

)xn, za |x| < 1,

pri £emu je(α

n

)=α(α− 1) · · · (α− n+ 1)

n!

Page 48: Kako zapisati rje²enja? - Math · Kako zapisati rje²enja? IMO zadaci i bodovne sheme ... Nemojte zaboraviti trivijalne slu£ajeve (nulu, prazan skup, bazu indukcije) Pi²ite obja²njenja,

ln (1 + x) =x

1− x2

2+x3

3− x4

4+ . . . =

∞∑n=1

(−1)n−1 xn

n, za |x| < 1

arctg x =x

1− x3

3+x5

5− x7

7+ . . . =

∞∑n=0

(−1)nx2n+1

2n+ 1, za |x| < 1

Zadaci

1. Koja je obi£na funkcija izvodnica niza zadanog formulom an = n2? Koja jeeksponencijalna funkcija izvodnica tog istog niza?

Rje²enje.

A(x) =∞∑n=0

n2xn = x2

∞∑n=2

n(n− 1)xn−2 + x∞∑n=1

nxn−1

= x2

∞∑n=0

(n+ 2)(n+ 1)︸ ︷︷ ︸2(n+2

2 )

xn + x

∞∑n=0

(n+ 1)xn

=2x2

(1− x)3+

x

(1− x)2=x(1 + x)

(1− x)3

E(x) =∞∑n=0

n2 xn

n!=∞∑n=1

nxn

(n− 1)!=∞∑n=1

((n− 1) + 1

)xn

(n− 1)!

=∞∑n=2

xn

(n− 2)!+∞∑n=1

xn

(n− 1)!= x2

∞∑n=0

xn

n!+ x

∞∑n=0

xn

n!= x(1 + x)ex.

2. Koliko ima permutacija skupa 1, 2, . . . , n koje imaju najvi²e jedan pad?Dakle, traºi se broj permutacija p1, p2, . . . , pn od 1, 2, . . . , n takvih da vrijedip1 < · · · < pk i pk+1 < . . . < pn za neki indeks k.

Rje²enje. Ozna£imo traºeni broj s an. O£igledno je a1 = 1, a2 = 2. Kasnije¢emo vidjeti da je prakti£no staviti a0 = 1.

Svaka permutacija od 1, 2, . . . , n, n+1 s opisanim svojstvom je ili identitetaili ima to£no jedan pad. Uzmemo li neku permutaciju od 1, 2, . . . , n, n+ 1 snajvi²e jednim padom i uklonimo element n+1, dobit ¢emo permutaciju od1, 2, . . . , n koja opet ima najvi²e jedan pad. Ako ta permutacija ima to£nojedan pad, tada broj n + 1 moºemo staviti na dva dozvoljena na£ina. Akoje pak ta permutacija identiteta, tada broj n+ 1 moºemo staviti bilo gdje,tj. na n+ 1 na£ina. Dobili smo

an+1 = 2(an − 1) + n+ 1,

tj. rekurzivnu relacijuan+1 = 2an + n− 1.

Pomnoºimo rekurziju s xn+1 i prosumirajmo po n ∈ N0.∞∑n=0

an+1xn+1 = 2x

∞∑n=0

anxn +

∞∑n=0

(n− 1)xn+1

Page 49: Kako zapisati rje²enja? - Math · Kako zapisati rje²enja? IMO zadaci i bodovne sheme ... Nemojte zaboraviti trivijalne slu£ajeve (nulu, prazan skup, bazu indukcije) Pi²ite obja²njenja,

∞∑n=1

anxn = 2x

∞∑n=0

anxn − x+ x3

∞∑n=0

(n+ 1)xn

Ozna£imo li funkciju izvodnicu s A(x), dobili smo (zbog a0 = 1):

A(x)− 1 = 2xA(x)− x(1− 2x)

(1− x)2,

tj.

A(x) =1

1− 2x− x

(1− x)2.

Rastav na parcijalne razlomke daje

A(x) =1

1− 2x+

1

1− x− 1

(1− x)2

te razvojem u red potencija dobivamo

A(x) =∞∑n=0

2nxn +∞∑n=0

xn −∞∑n=0

(n+ 1)xn =∞∑n=0

(2n − n)xn.

Zato je kona£noan = [xn]A(x) = 2n − n.

3. Na koliko na£ina se nov£anica od 10 kuna moºe razmijeniti u kovanice?

Rje²enje. Pretvorimo sve iznose u lipe, kako bi bili cjelobrojni. Zapravotraºimo koecijent uz x1000 u produktu

f(x) = (1 + x+ x2 + x3 + · · · )︸ ︷︷ ︸kovanice od 1 lp

(1 + x2 + x4 + x6 + · · · )︸ ︷︷ ︸kovanice od 2 lp

· (1 + x5 + x10 + x15 + · · · )︸ ︷︷ ︸kovanice od 5 lp

(1 + x10 + x20 + x30 + · · · )︸ ︷︷ ︸kovanice od 10 lp

· (1 + x20 + x40 + x60 + · · · )︸ ︷︷ ︸kovanice od 20 lp

(1 + x50 + x100 + x150 + · · · )︸ ︷︷ ︸kovanice od 50 lp

· (1 + x100 + x200 + x300 + · · · )︸ ︷︷ ︸kovanice od 1 kn

(1 + x200 + x400 + x600 + · · · )︸ ︷︷ ︸kovanice od 2 kn

· (1 + x500 + x1000 + x1500 + · · · )︸ ︷︷ ︸kovanice od 5 kn

,

koji se izra£una kao

(1− x)−1(1− x2)−1(1− x5)−1(1− x10)−1(1− x20)−1

· (1− x50)−1(1− x100)−1(1− x200)−1(1− x500)−1.

Ako ºelimo razviti f(x) u red potencija do £lana x1000, moºemo u Mathe-matici koristiti naredbu

Series[f [x], x, 0, 1000] .

Ovdje 0 naprosto zna£i da se razvija oko nule, tj. u potencije (x−0)n = xn,²to ¢emo mi uvijek raditi. O£itavanje koecijenta uz x1000 daje

[x1000]f(x) = 327631321.

Page 50: Kako zapisati rje²enja? - Math · Kako zapisati rje²enja? IMO zadaci i bodovne sheme ... Nemojte zaboraviti trivijalne slu£ajeve (nulu, prazan skup, bazu indukcije) Pi²ite obja²njenja,

4. (Rumunjska 2003.) Koliko n-znamenkastih brojeva sa znamenkama iz skupa2, 3, 7, 9 je djeljivo s 3?

Rje²enje iz [1]. Koristit ¢emo karakterizaciju da je broj djeljiv s 3 ako isamo ako je zbroj njegovih znamenaka djeljiv s 3.

Promotrimo funkciju izvodnicu

f(x) := (x2 + x3 + x7 + x9)n.

Njen koecijent uz xm nam govori koliko ima n-znamenkastih brojeva saznamenkama iz skupa 2, 3, 7, 9 £iji zbroj znamenaka je upravom. Uzmimosada ω = e2πi/3 = −1

2+ i

√3

2, tj. ω je primitivni kompleksni tre¢i korijen iz

jedinice. Primijetimo da za m ∈ Z vrijedi

1 + ωm + ω2m =

3 ako je m djeljiv s p,

1−ω3m

1−ωm = 0 ako m nije djeljiv s 3.

Pogledajmo £emu je jednako

1

3

(f(1) + f(ω) + f(ω2)

).

Ako raspi²emo f(x) =∑9n

m=0 amxm, prema gornjoj formuli to je

9n∑m=0

am1

3(1 + ωm + ω2m) =

∑0≤m≤9n

m je djeljiv s 3

am,

²to je upravo traºeni broj.

S druge pak strane, direktno uvr²tavanje daje

1

3

(f(1) + f(ω) + f(ω2)

)=

1

3

(4n + (ω2 + ω3 + ω7 + ω9)n + (ω4 + ω6 + ω14 + ω18)n

)=

1

3

(4n + 2(2 + ω + ω2︸ ︷︷ ︸

1+0=1

)n)

=4n + 2

3

i to je ºeljeni rezultat.

5. (IMO 1995. #6 ) Neka je p ≥ 3 prost broj. Naite broj svih p-£lanihpodskupova od 1, 2, . . . , 2p− 1, 2p £iji zbroj elemenata je djeljiv s p.

Rje²enje iz knjige [4]. Promotrimo funkciju izvodnicu

f(x, y) :=

2p∏j=1

(xyj − 1).

Za y = 1 odmah vidimo da f(x, 1) do na predznak prebrojava sve pod-skupove od 1, . . . , 2p, ali zanimljivo je uvrstiti i druge vrijednosti za y.

Page 51: Kako zapisati rje²enja? - Math · Kako zapisati rje²enja? IMO zadaci i bodovne sheme ... Nemojte zaboraviti trivijalne slu£ajeve (nulu, prazan skup, bazu indukcije) Pi²ite obja²njenja,

Ozna£imo ω = e2πi/p, tj. ω je primitivni kompleksni p-ti korijen iz jedinice.Primijetimo da za m ∈ Z vrijedi

p−1∑k=0

ωkm =

p ako je m djeljiv s p,

1−ωpm1−ωn = 0 ako m nije djeljiv s p.

Pogledajmo sada ²to prebraja funkcija izvodnica

1

p

p−1∑k=0

f(x, ωk).

Njezin koecijent uz xp je

−∑

S⊆1,...,2p|S|=p

1

p

p−1∑k=0

ωk(zbroj elemenata od S) = −∑

S⊆1,...,2p|S|=p

zbroj elemenata od S je djeljiv s p

1

pa nakon mnoºenja sa −1 dobivamo upravo broj koji se traºi u zadatku.

S druge strane, za k ∈ 1, 2, . . . , p − 1 imamo da su k, 2k, . . . , (p − 1)kmodulo p naprosto ispermutirani ostaci 1, 2 . . . , p− 1, ²to nam daje

f(x, ωk) =

2p∏j=1

(xωjk − 1) =( p−1∏j=0

(xωj − 1))2

=(ωp(p−1)/2

p−1∏j=0

(x− ω−j))2

=( p−1∏j=0

(x− ω−j))2

= (xp − 1)2,

pri £emu smo koristili faktorizaciju polinoma pomo¢u njegovih kompleksnihnulto£aka. Dakle,

1

p

p−1∑k=0

f(x, ωk) =1

p(x− 1)2p +

p− 1

p(xp − 1)2,

a koecijent posljednje funkcije izvodnice uz xn iznosi

−1

p

(2p

p

)+p− 1

p· (−2).

Slijedi da je traºeni broj jednak(2pp

)+ 2p− 2

p.

6. (Euler) Dokaºite da je broj rastava prirodnog broja na razli£ite pribrojnikejednak broju rastava prirodnog broja na neparne pribrojnike.

Rje²enje. Ozna£imo:

Rn = broj rastava od n na razli£ite pribrojnike,Nn = broj rastava od n na neparne pribrojnike.

Page 52: Kako zapisati rje²enja? - Math · Kako zapisati rje²enja? IMO zadaci i bodovne sheme ... Nemojte zaboraviti trivijalne slu£ajeve (nulu, prazan skup, bazu indukcije) Pi²ite obja²njenja,

∞∑n=0

Rnxn = (1 + x)(1 + x2)(1 + x3) · · · =

∞∏k=1

(1 + xk) =∞∏k=1

1− x2k

1− xk

=

∞∏k=1

(1− x2k)

∞∏k=1

(1− xk)=

1∏k∈N

k neparan

(1− xk)=

∏k∈N

k neparan

1

1− xk

=∏k∈N

k neparan

(1 + xk + x2k + x3k + · · · ) =∞∑n=0

Nnxn

Izjedna£avanjem koecijenata slijedi Rn = Nn.

7. (Zadatak iz knjige [5], poglavlje 2 ) Neka je Dn broj permutacija skupa1, 2, . . . , n bez ksnih to£aka, tj. takvih bijekcija

f : 1, 2, . . . , n → 1, 2, . . . , n

da za svaki x vrijedi f(x) 6= x. esto se kaºe da je Dn broj deranºmanan-£lanog skupa. Jo² stavljamo D0 := 1. Najprije izvedite formulu za eks-ponencijalnu funkciju izvodnicu niza (Dn)∞n=0, a potom dokaºite formulu

Dn = n!n∑k=0

(−1)k

k!.

Rje²enje. Svaka permutacija od 1, 2, . . . , n ksira nekih k elemenata tepotom deranºira preostalih n − k elemenata. Kako se k-£lani podskupmoºe odabrati na

(nk

)na£ina, dobili smo formulu

n! =n∑k=0

(n

k

)Dn−k.

Mnoºenjem s xn

n!i zbrajanjem dobivamo

∞∑n=0

xn =∞∑n=0

n∑k=0

xk

k!

Dn−kxn−k

(n− k)!.

Na desnoj strani moºemo prepoznati produkt redova po principu svaki sasvakim: ( ∞∑

k=0

xk

k!︸ ︷︷ ︸=ex

)( ∞∑m=0

Dmxm

m!

).

Ozna£imo li eksponencijalnu funkciju izvodnicu niza s D(x), dobili smo

1

1− x= exD(x), tj. D(x) =

e−x

1− x.

Razvijmo sada ponovno D(x) u red.

D(x) =( ∞∑k=0

(−1)kxk

k!

)(1 + x+ x2 + x3 + · · ·

).

Page 53: Kako zapisati rje²enja? - Math · Kako zapisati rje²enja? IMO zadaci i bodovne sheme ... Nemojte zaboraviti trivijalne slu£ajeve (nulu, prazan skup, bazu indukcije) Pi²ite obja²njenja,

Usporeivanje koecijenata uz xn daje

Dn

n!=

n∑k=0

(−1)k

k!.

8. (Zadatak iz knjige [5], poglavlje 1 ) Neka je Bn broj particija skupa od nelemenata, tzv. Bellov broj. Tako je npr.

B0 := 1, B1 = 1, B2 = 2, B3 = 5, B4 = 15, B5 = 52.

Najprije izvedite formulu za eksponencijalnu funkciju izvodnicu tog niza, apotom dokaºite formulu

Bn =1

e

∞∑k=0

kn

k!.

Rje²enje. Diskutiranjem u kojem £lanu particije se nalazi broj n + 1 iozna£avaju¢i s k broj preostalih elemenata u tom £lanu dobivamo rekurzivnurelaciju

Bn+1 =n∑k=0

(n

k

)Bk.

Mnoºenjem s xn

n!i zbrajanjem dobivamo

∞∑n=0

Bn+1xn

n!=∞∑n=0

n∑k=0

xn−k

(n− k)!

Bkxk

k!.

Na lijevoj strani prepoznajemo derivaciju:

d

dx

∞∑n=0

Bnxn

n!=∞∑n=1

Bnxn−1

(n− 1)!,

dok na desnoj prepoznajemo produkt redova po principu svaki sa svakim:( ∞∑m=0

xm

m!︸ ︷︷ ︸=ex

)( ∞∑k=0

Bkxk

k!

).

Ozna£imo li eksponencijalnu funkciju izvodnicu niza s B(x), dobili smo

B′(x) = exB(x).

Sada rje²avamo tu diferencijalmu jednadºbu:

d

dx(lnB(x)) =

B′(x)

B(x)= ex

lnB(x) = ex + C

B(x) = eex+C

Konstanta C se odredi iz

1 = B0 = B(0) = e1+C , tj. C = −1.

Page 54: Kako zapisati rje²enja? - Math · Kako zapisati rje²enja? IMO zadaci i bodovne sheme ... Nemojte zaboraviti trivijalne slu£ajeve (nulu, prazan skup, bazu indukcije) Pi²ite obja²njenja,

Dakle,B(x) = ee

x−1.

Iskoristimo sada dvaput razvoj eksponencijalne funkcije u red:

B(x) =1

eeex

=1

e

∞∑k=0

ekx

k!=

1

e

∞∑k=0

∞∑n=0

(kx)n

k!n!=

1

e

∞∑n=0

( ∞∑k=0

kn

k!

)xnn!.

Usporeivanje koecijenata s B(x) =∑∞

n=0Bnxn

n!daje traºenu formulu.

Asimptotika nizova

esto je potrebno ustanoviti asimptotsko pona²anje (npr. brzinu rasta) nekogkombinatorno dobivenog niza, a funkcije izvodnice mogu pomo¢i kod toga.

Kaºemo da su nizovi (an)∞n=0 i (bn)∞n=0 asimptotski jednaki i pi²emo an ∼ bn akovrijedi

limn→∞

anbn

= 1.

Naprimjer, poznata Stirlingova formula se moºe zapisati

n! ∼√

2πn(ne

)n.

Sljede¢i teorem je samo malo neprecizan jer ºelimo izbje¢i previ²e tehni£ke poj-move.

Teorem 12. (Iz £lanka [3]; vidjeti knjigu [2], poglavlje 11 ) Neka je (an)∞n=0 kombi-natorno dobiveni niz i neka njegova obi£na funkcija izvodnica A(x) =

∑∞n=0 anx

n

konvergira na intervalu [−r, r〉 za neki r > 0. Pretpostavimo da je ona oblikaA(x) = f(x)g(x) + h(x), pri £emu vrijedi:

• f(x) =(− ln(1− x

r))b

(1− xr)c, c 6∈ N i nije ba² b = c = 0,

• postoji limes L := limx↑r

g(x) ∈ R \ 0,

• postoji limes limx↑r

h(x) ∈ R.

Ako je c 6= 0, tada vrijedi

an ∼L(lnn)b(1/r)n

nc+1Γ(−c),

a ako je c = 0, tada vrijedi

an ∼bL(lnn)b−1(1/r)n

n.

Ovdje Γ ozna£ava gama-funkciju. Obi£no je dovoljno znati da je Γ(k) = (k − 1)!za k ∈ N te Γ(1/2) =

√π, Γ(−1/2) = −2

√π, Γ(3/2) =

√π/2. Primijetimo jo²

da je x = r prvi singularitet funkcije A(x).

Page 55: Kako zapisati rje²enja? - Math · Kako zapisati rje²enja? IMO zadaci i bodovne sheme ... Nemojte zaboraviti trivijalne slu£ajeve (nulu, prazan skup, bazu indukcije) Pi²ite obja²njenja,

Zadaci

9. Ako je Dn broj permutacija n-£lanog skupa bez ksnih to£aka, dokaºite

Dn ∼n!

e.

Rje²enje. Funkcija izvodnica od Dn/n! je

D(x) = (1− x)−1e−x

pa primjenjujemo teorem za r = 1, b = 0, c = −1, g(x) = e−x, h(x) = 0,

L = limx↑1

e−x = e−1.

DobivamoDn

n!∼ L 1n

n0 Γ(1)= L =

1

e.

Napomenimo da se isti zaklju£ak moºe izvesti i iz op¢e formule niza:

limn→∞

Dn

n!= lim

n→∞

n∑k=0

(−1)k

k!=∞∑k=0

(−1)k

k!= e−1.

Osim toga, ovdje nije te²ko dokazati i precizniju tvrdnju: Dn je najbliºicijeli broj broju n!/e.

10. (Primjer iz knjige [2], poglavlje 11 ) Neka je Cn broj na£ina na koje se moºeizra£unati izraz a1 ∗ a2 ∗ · · · ∗ an (koji ima n− 1 znakova ∗) ukoliko je vaºanporedak vr²enja operacije ∗. Naime, zagradama moºemo odrediti redoslijedizv²avanja pa je npr. C4 = 5, jer su mogu¢nosti

a1 ∗ (a2 ∗ (a3 ∗ a4)), a1 ∗ ((a2 ∗ a3) ∗ a4), (a1 ∗ a2) ∗ (a3 ∗ a4),(a1 ∗ (a2 ∗ a3)) ∗ a4, ((a1 ∗ a2) ∗ a3) ∗ a4.

To su tzv. Catalanovi brojevi (pomaknuti za 1) i stavljamo C0 = 0. Izvediteizraz za obi£nu funkciju izvodnicu niza (Cn)∞n=0 pa potom odredite njegovuasimptotiku.

Rje²enje. U izrazu a1∗a2∗· · ·∗an promotrimo instancu od ∗ koja se izvr²avaposljednja. Neka lijevo od nje ima k varijabli i desno od nje n− k varijabli.Dobivamo rekurzivnu relaciju

Cn =n−1∑k=1

CkCn−k.

Pomnoºimo je s xn i prosumirajmo po n ≥ 2.

∞∑n=2

Cnxn =

∞∑n=2

n−1∑k=1

CkxkCn−kx

n−k =∞∑

k,m=1

CkxkCmx

m.

Ozna£imo li funkciju izvodnicu C(x) :=∑∞

n=0Cnxn, dobivamo (zbog C1 =

1):C(x)− x = C(x)2,

Page 56: Kako zapisati rje²enja? - Math · Kako zapisati rje²enja? IMO zadaci i bodovne sheme ... Nemojte zaboraviti trivijalne slu£ajeve (nulu, prazan skup, bazu indukcije) Pi²ite obja²njenja,

a rje²avanje kvadratne jednadºbe po C(x) daje

C(x) =1−√

1− 4x

2.

Kako funkciju izvodnicu moºemo zapisati

C(x) =(

1− x

1/4

)1/2(− 1

2

)+

1

2,

moºemo primijeniti prethodni teorem uz r = 1/4, b = 0, c = 1/2, g(x) =−1/2, L = −1/2, h(x) = 1/2 pa imamo

Cn ∼(−1/2) 4n

n3/2 Γ(−1/2)=

4n−1

√π n3/2

.

To je traºena asimptotika niza.

Napomenimo da se razvojem od C(x) u red moºe lako izvesti i poznataop¢a formula

Cn =1

n

(2n− 2

n− 1

), za n ∈ N.

Zadaci za samostalan rad

1. Mea ºeli olimpijskoj ekipi podijeliti 20 jednakih £okolada. Ilko je postaviosljede¢e uvjete na raspodjelu £okolada.

• Adrian, Daniel i Erik mogu dobiti najvi²e po jednu £okoladu svaki.

• Mislav i Petar moraju dobiti neparan broj £okolada.

• Vlatka mora dobiti paran broj £okolada i to barem 2 £okolade.

Na koliko na£ina Mea moºe podijeliti te £okolade?

Rje²enje. Traºimo koecijent uz x20 nakon ²to se izmnoºi produkt

f(x) = (1 + x)3( ∞∑k=0

x2k+1)2( ∞∑

k=0

x2k+2).

Izra£unajmo zatvorenu formulu za f(x).

f(x) = (1 + x)3x4( ∞∑k=0

x2k)3

= (1 + x)3x4( 1

1− x2

)3

= (1 + x)3x4 1

(1− x)3(1 + x)3=

x4

(1− x)3.

Nadalje, razvoj u red potencija koriste¢i jednu od standardnih formula daje

f(x) = x4

∞∑n=0

(n+ 2

2

)xn =

∞∑n=0

(n+ 2)(n+ 1)

2xn+4.

lan s potencijom x20 odgovara indeksu n = 16 pa moºemo o£itati koeci-jent:

[x20]f(x) =18 · 17

2= 153.

Dakle, Mea moºe raspodijeliti £okolade na 153 na£ina.

Page 57: Kako zapisati rje²enja? - Math · Kako zapisati rje²enja? IMO zadaci i bodovne sheme ... Nemojte zaboraviti trivijalne slu£ajeve (nulu, prazan skup, bazu indukcije) Pi²ite obja²njenja,

2. (Rezultat iz knjige [5], poglavlje 4 ) Pretpostavimo da je skup nenegativnihcijelih brojeva particioniran na k beskona£nih aritmeti£kih nizova s razli-kama d1, d2, . . . , dk, pri £emu je k ≥ 2 prirodan broj i d1 ≥ d2 ≥ . . . ≥ dk.Dokaºite da mora vrijediti

∑kj=1

1dj

= 1 i d1 = d2.

Rje²enje. Neka su aritmeti£ki nizovi na koje je N0 rastavljen dani sa (aj +mdj)

∞m=0, j = 1, 2, . . . , k. Iz £injenice da oni particioniraju N0 slijedi

∞∑n=0

xn =k∑j=1

∞∑m=0

xaj+mdj ,

a sumiranje redova daje

1

1− x=

k∑j=1

xaj

1− xdj.

Pomnoºimo gornju jednakost s 1− x i pustimo limes kada x→ 1. Zbog

limx→1

1− x1− xdj

= limx→1

1

1 + x+ x2 + · · ·+ xdj−1=

1

dj

dobivamo

1 =k∑j=1

1

dj.

Zbog k ≥ 2 mora biti dj ≥ 2 za svaki j. Pretpostavimo suprotno tvrdnjid1 = d2, da vrijedi d1 > d2 ≥ . . . ≥ dk. Rastavimo desnu stranu gornjejednakosti racionalnih funkcija na parcijalne razlomke nad poljem C. Kodrastava od

xa1

1− xd1se pojavljuje £lan

A

x− e2πi/d1, A 6= 0, koji se ne pojavljuje

na lijevoj strani, a ne moºe se ni pokratiti s drugim parcijalnim razlomcimana desnoj strani jer e2πi/d1 nije nulto£ka nijednog od polinoma xdj − 1,j = 2, . . . , k. To je kontradikcija.

3. (Primjer iz knjige [5], poglavlje 4 ) Za cijeli broj n ≥ 0 izra£unajte

n∑k=0

(n+ k

2k

)2n−k.

Rezultat zapi²ite kao zatvorenu formulu, bez suma i produkata.

Rje²enje. Ozna£imo an :=∑n

k=0

(n+k2k

)2n−k i neka je A(x) =

∑n=0 anx

n

funkcija izvodnica tog niza. Kori²tenjem standardnih formula za sume re-dova potencija moºemo ra£unati

A(x) =∞∑n=0

n∑k=0

(n+ k

2k

)2n−kxn =

∞∑k=0

∞∑n=k

(n− k + 2k

2k

)(2x)n−kxk

=∞∑k=0

xk∞∑n=0

(n+ 2k

2k

)(2x)n =

∞∑k=0

xk1

(1− 2x)2k+1

=1

1− 2x

∞∑k=0

( x

(1− 2x)2

)k=

1

1− 2x

1

1− x(1−2x)2

Page 58: Kako zapisati rje²enja? - Math · Kako zapisati rje²enja? IMO zadaci i bodovne sheme ... Nemojte zaboraviti trivijalne slu£ajeve (nulu, prazan skup, bazu indukcije) Pi²ite obja²njenja,

=1− 2x

1− 5x+ 4x2=

1− 2x

(1− x)(1− 4x).

Rastav na parcijalne razlomke je

A(x) =1

3· 1

1− x+

2

3· 1

1− 4x,

a potom razvijamo u red potencija koriste¢i standardne formule:

A(x) =1

3

∞∑n=0

xn +2

3

∞∑n=0

(4x)n =∞∑n=0

22n+1 + 1

3xn.

Usporeivanje koecijenata uz xn daje

an = [xn]A(x) =22n+1 + 1

3.

4. (Primjer iz knjige [2], poglavlje 11 ) Promotrimo izraz 0 ∧ 0 ∧ · · · ∧ 0, ukojem ima n nula i n − 1 znakova potenciranja ∧. Otprije znamo da seredoslijed izvr²avanja operacija ∧ moºe izvesti na Cn na£ina, pri £emu jeCn (pomaknuti) Catalanov broj. Neka je Zn broj takvih evaluacija kojekao rezultat daju 0. Pritom smatramo da vrijedi: 0 ∧ 0 = 1, 0 ∧ 1 = 0,1 ∧ 0 = 1, 1 ∧ 1 = 1. Dokaºite da je lim

n→∞ZnCn

= 5−√

510

.

Napomena: Naprimjer, Z3 = 1 jer je (0∧ 0)∧ 0 = 1∧ 0 = 1 i 0∧ (0∧ 0) =0 ∧ 1 = 0.

Rje²enje. Prakti£no nam je staviti Z0 = 0, a o£igledno je Z1 = 1. Svakaevaluacija koja daje vrijednost 0 mora biti oblika A ∧ B, pri £emu Apredstavlja formulu s k nula koja se evaluira u 0, a B predstavlja formulu sn− k nula koja se evaluira u 1. Na taj na£in dobivamo rekurzivnu relaciju

Zn =n−1∑k=1

Zk (Cn−k − Zn−k).

Pomnoºimo je s xn i prosumirajmo po n ≥ 2.

∞∑n=2

Znxn =

∞∑n=2

n−1∑k=1

Zkxk(Cn−k − Zn−k)xn−k

=( ∞∑k=1

Zkxk)( ∞∑

m=1

(Cm − Zm)xm)

Ozna£imo sa Z(x) funkciju izvodnicu od (Zn)∞n=0, a ve¢ prije smo s C(x)ozna£ili funkciju izvodnicu niza (Cn)∞n=0. Dobili smo kvadratnu jednadºbupo Z(x),

Z(x)− x = Z(x)(C(x)− Z(x)

),

£ije rje²avanje daje

Z(x) = −1

2

(1− C(x)

)+

1

2

√(1− C(x)

)2+ 4x.

Page 59: Kako zapisati rje²enja? - Math · Kako zapisati rje²enja? IMO zadaci i bodovne sheme ... Nemojte zaboraviti trivijalne slu£ajeve (nulu, prazan skup, bazu indukcije) Pi²ite obja²njenja,

Odabrali smo pozitivni predznak ispred korijena jer mora biti Z(0) = Z0 =0.

Ve¢ smo bili izra£unali

C(x) =1

2− 1

2

√1− 4x.

Prvi singularitet funkcije Z(x) je ili r = 14ili rje²enje jednadºbe

(1 −

C(x))2

+ 4x = 0. Posljednja jednadºba ima rje²enje x = −49, ali je 4

9> 1

4

pa ostajemo kod r = 14. Slutimo da se Z(x) moºe prikazati u obliku

Z(x) = (1− 4x)1/2g(x) + Z(14).

Primijetimo da jeC(1

4) = 1

2, Z(1

4) =

√5−14.

Trebamo izra£unati

L = limx↑1/4

g(x) = limx↑1/4

Z(x)− Z(14)

(1− 4x)1/2.

Supstitucijom

s =√

1− 4x, x = (1− s2)/4, C(x) = (1− s)/2

taj limes postaje

L = lims↓0

−(1 + s)/2 +√

(1 + s)2/4 + (1− s2)− 2Z(14)

2s

= lims↓0

−s+√

5 + 2s− 3s2 −√

5

4s= lim

s↓0

−s+ 2s−3s2√5+2s−3s2+

√5

4s

= lims↓0

−1 + 2−3s√5+2s−3s2+

√5

4=−1 + 2

2√

5

4= −5−

√5

20.

Kona£no moºemo primijeniti teorem o asimptotici nizova uz parametre

r = 1/4, b = 0, c = 1/2, h(x) = Z(14)

i dobiti

Zn ∼L 4n

n3/2 Γ(−1/2)=

5−√

5

40

4n

n3/2√π.

Dijeljenje s formulom izvedenom na satu,

Cn ∼4n−1

n3/2√π,

dajeZnCn∼ 5−

√5

10.

Page 60: Kako zapisati rje²enja? - Math · Kako zapisati rje²enja? IMO zadaci i bodovne sheme ... Nemojte zaboraviti trivijalne slu£ajeve (nulu, prazan skup, bazu indukcije) Pi²ite obja²njenja,

Kori²tena literatura i dodatni izvori zadataka

[1] Z. R. Abel, Multivariate Generating Functions and Other Tidbits, Mathe-matical reections 2, 2006.

[2] E. A. Bender, S. G. Williamson, Foundations of Combinatorics with Appli-cations, Dover, 2006.

[3] P. Flajolet, A. Odlyzko, Singularity Analysis of Generating Functions, SIAMJ. Disc. Math. 3 (1990), 216240.

[4] . Hanj², Meunarodne matemati£ke olimpijade, tre¢e izdanje, Element,Zagreb, 2000.

[5] H. S. Wilf, generatingfunctionology, A K Peters/CRC Press, tre¢e izdanje,2005.

Page 61: Kako zapisati rje²enja? - Math · Kako zapisati rje²enja? IMO zadaci i bodovne sheme ... Nemojte zaboraviti trivijalne slu£ajeve (nulu, prazan skup, bazu indukcije) Pi²ite obja²njenja,

3. Fourierova analiza

Fourierova analiza je posebno korisna kod kombinatornih problema koji se bavearitmeti£kim strukturama, tj. kada uz same skupove promatramo i operaciju zbra-janja, a ponekad i operaciju mnoºenja. Kako bismo izlaganje odrºali elementar-nim, ovdje ¢emo raditi samo na kona£nim komutativnim grupama.

Za prirodni broj d ozna£imo sa Zd skup ostataka pri dijeljenju s d, tj.

Zd := 0, 1, 2, 3, . . . , d− 2, d− 1.

Neka se na skupu Zd podrazumijevaju operacije zbrajanja i mnoºenja modulo d,tj. nakon izvr²enog zbrajanja ili mnoºenja jo² podijelimo rezultat s d i uzmemosamo ostatak pri dijeljenju. Tako npr. na Z4 imamo tablice zbrajanja i mnoºenja:

+ 0 1 2 30 0 1 2 31 1 2 3 02 2 3 0 13 3 0 1 2

· 0 1 2 30 0 0 0 01 0 1 2 32 0 2 0 23 0 3 2 1

Fiksirajmo sada n ∈ N i d1, . . . , dn ∈ N \ 1. Kada pi²emo

A = Zd1 ⊕ Zd2 ⊕ · · · ⊕ Zdn ,

tada smatramo da je A Kartezijev produkt skupova Zd1 , . . . ,Zdn na kojem sepromatra zbrajanje po koordinatama. Drugim rije£ima, elementi od A su n-torkex = (x1, x2, . . . , xn) koje se zbrajaju kao:

(x1, x2, . . . , xn) + (y1, y2, . . . , yn) := (x1 + y1, x2 + y2, . . . , xn + yn).

esto pi²emo samo 0 umjesto n-torke samih nula, (0, 0, . . . , 0). Tako npr. opera-cija zbrajanja na Z2 ⊕ Z2 glasi:

+ (0,0) (0,1) (1,0) (1,1)(0,0) (0,0) (0,1) (1,0) (1,1)(0,1) (0,1) (0,0) (1,1) (1,0)(1,0) (1,0) (1,1) (0,0) (0,1)(1,1) (1,1) (1,0) (0,1) (0,0)

Kaºemo da je (A,+) primjer kona£ne komutativne grupe i (preciznije) da je Adirektna suma kona£no mnogo kona£nih cikli£kih grupa. Zapravo, svaka kona£nakomutativna grupa ima strukturu kao gornji primjer za odreene parametre n id1, . . . , dn.

Denirajmo preslikavanje

E : A× A→ C, E(x, ξ) :=(e2πi/d1

)x1ξ1(e2πi/d2)x2ξ2 · · · (e2πi/dn

)xnξnza x = (x1, x2, . . . , xn), ξ = (ξ1, ξ2, . . . , ξn) iz A.

Primijetimo da E poprima vrijednosti u kompleksnim brojevima modula 1. Jedinirazlog za²to lijevi argument pi²emo latini£nim slovom x, a desni gr£kim slovom

Page 62: Kako zapisati rje²enja? - Math · Kako zapisati rje²enja? IMO zadaci i bodovne sheme ... Nemojte zaboraviti trivijalne slu£ajeve (nulu, prazan skup, bazu indukcije) Pi²ite obja²njenja,

ξ, je kako bismo naglasili da oni ºive u dvije razli£ite kopije od A, povezanepreslikavanjem E.

To preslikavanje ima sljede¢a o£igledna svojstva.

E(x+ y, ξ) = E(x, ξ)E(y, ξ), E(x, ξ + ζ) = E(x, ξ)E(x, ζ),

E(0, ξ) = 1, E(x,0) = 1, E(−x, ξ) = E(x, ξ), E(x,−ξ) = E(x, ξ),∑x∈A

E(x, ξ) =

|A| ako je ξ = 00 ako je ξ 6= 0

∑ξ∈A

E(x, ξ) =

|A| ako je x = 00 ako je x 6= 0

Naprimjer, posljednja formula slijedi za x = (x1, x2, . . . , xn), ξ = (ξ1, ξ2, . . . , ξn)iz jednakosti

∑ξ∈A

E(x, ξ) =

( d1−1∑ξ1=0

(e2πix1/d1

)ξ1) · · ·( dn−1∑ξn=0

(e2πixn/dn

)ξn).

Preostaje primijetiti da za xj 6= 0 vrijedi e2πixj/dj 6= 1 pa formula za parcijalnusumu geometrijskog reda daje

dj−1∑ξj=0

(e2πixj/dj

)ξj =1− e2πixj

1− e2πixj/dj=

1− 1

1− e2πixj/dj= 0,

dok za xj = 0 imamo

dj−1∑ξj=0

(e2πixj/dj

)ξj =

dj−1∑ξj=0

1 = dj.

Produkt preºivi samo kada je x1 = · · · = xn = 0 i tada je jednak d1 · · · dn = |A|.

Zadaci

1. Koliko se najvi²e moºe odabrati meusobno okomitih vektora iz skupa−1, 11024?Napomena: Vektori u, v ∈ Rn, u = (u1, . . . , un), v = (v1, . . . , vn) su okomitiako i samo ako je njihov skalarni produkt jednak 0, tj. vrijedi u · v =∑n

j=1 ujvj = 0.

Rje²enje. U prostoru Rn moºe postojati najvi²e n u parovima okomitih ne-nul vektora. Zato ¢e u na²em zadatku odgovor biti 1024 ako jo² pronaemoprimjer koji ima to£no 1024 vektora.

Promotrimo grupu A = Z102 = Z2 ⊕ · · · ⊕ Z2︸ ︷︷ ︸

10

. O£igledno A ima 210 = 1024

elemenata. Za svaki x ∈ A promotrimo vektor

vx :=(E(x, ξ) : ξ ∈ A

)duljine |A| = 1024. Kako se u formuli za E sada pojavljuju samo drugi ko-rijeni iz jedinice (tj. samo potencije od −1), zaklju£ujemo da sve koordinate

Page 63: Kako zapisati rje²enja? - Math · Kako zapisati rje²enja? IMO zadaci i bodovne sheme ... Nemojte zaboraviti trivijalne slu£ajeve (nulu, prazan skup, bazu indukcije) Pi²ite obja²njenja,

vektora pripadaju skupu −1, 1. Iz svojstava od E se odmah vidi da suvektori vx meusobno okomiti:

vx · vy =∑ξ∈A

E(x, ξ)E(y, ξ) =∑ξ∈A

E(x, ξ)E(y, ξ) =∑ξ∈A

E(x− y, ξ) = 0

ako je x 6= y.

Kona£no moºemo uvesti klju£ni pojam Fourierove analize. Fourierova transfor-macija funkcije f : A→ C je nova funkcija f : A→ C denirana formulom

f(ξ) :=∑x∈A

f(x)E(x, ξ) za svaki ξ ∈ A.

Naprimjer:

(a) Ako je A = Z4 i ako funkcije na A pi²emo kao ureene £etvorke

f = (f0, f1, f2, f3),

tada je

f =(f0 +f1 +f2 +f3, f0 +if1−f2−if3, f0−f1 +f2−f3, f0−if1−f2 +if3

).

(b) Ako je A = Z2 ⊕ Z2 i ako funkcije na A pi²emo kao ureene £etvorke

f = (f00, f01, f10, f11),

tada je

f =(f00 + f01 + f10 + f11, f00 − f01 + f10 − f11,

f00 + f01 − f10 − f11, f00 − f01 − f10 + f11

).

Neka svojstva Fourierove transformacije su dana u sljede¢em teoremu.

Teorem 13. (a) Vrijedi sljede¢a formula inverzije:∑ξ∈A

f(ξ)E(x, ξ) = |A|f(x) za svaki x ∈ A.

Njom se polazna funkcija f moºe rekonstruirati iz svoje Fourierove tran-sformacije.Posebno, Fourierova transformacija je injektivna, tj. f = g implicira f = g.

(b) Vrijedi Plancherelov identitet:∑ξ∈A

|f(ξ)|2 = |A|∑x∈A

|f(x)|2

i op¢enitije: ∑ξ∈A

f(ξ)g(ξ) = |A|∑x∈A

f(x)g(x).

Page 64: Kako zapisati rje²enja? - Math · Kako zapisati rje²enja? IMO zadaci i bodovne sheme ... Nemojte zaboraviti trivijalne slu£ajeve (nulu, prazan skup, bazu indukcije) Pi²ite obja²njenja,

(c) Ako je funkcija h denirana kao tzv. konvolucija od f i g,

h(x) :=∑y∈A

f(x− y)g(y) za svaki x ∈ A,

²to pi²emo h = f ∗ g, tada vrijedi

h(ξ) = f(ξ)g(ξ) za svaki ξ ∈ A.

Drugim rije£ima, Fourierova transformacija prevodi konvoluciju u obi£niprodukt.

(d) Fourierova transformacija je linearna, tj.

αf + βg = αf + βg.

Dokaz. (a) ∑ξ∈A

f(ξ)E(x, ξ) =∑ξ∈A

∑y∈A

f(y)E(y, ξ)E(x, ξ)

=∑y∈A

f(y)∑ξ∈A

E(y − x, ξ) = |A|f(x)

(b) ∑ξ∈A

f(ξ)g(ξ) =∑ξ∈A

∑x,y∈A

f(x)g(y)E(x, ξ)E(y, ξ)

=∑x,y∈A

f(x)g(y)∑ξ∈A

E(x− y, ξ) = |A|∑x∈A

f(x)g(x)

(c)

h(ξ) =∑x∈A

(∑y∈A

f(x− y)g(y))E(x, ξ)

=∑x,y∈A

f(x− y)E(x− y, ξ) g(y)E(y, ξ)

=∑z,y∈A

f(z)E(z, ξ) g(y)E(y, ξ)

=(∑z∈A

f(z)E(z, ξ))(∑

y∈A

g(y)E(y, ξ))

= f(ξ)g(ξ)

(d)

( αf + βg)(ξ) =∑x∈A

(αf(x) + βg(x)

)E(x, ξ)

= α∑x∈A

f(x)E(x, ξ) + β∑x∈A

g(x)E(x, ξ)

= αf(ξ) + βg(ξ)

Page 65: Kako zapisati rje²enja? - Math · Kako zapisati rje²enja? IMO zadaci i bodovne sheme ... Nemojte zaboraviti trivijalne slu£ajeve (nulu, prazan skup, bazu indukcije) Pi²ite obja²njenja,

Zadaci

2. (Bourgain, preuzet iz knjige [2], poglavlje 4 ) Neka je p prost broj i S ⊆1, 2, 3, . . . , p− 1 takav da je |S| > p3/4. Dokaºite da za svaki cijeli broj mpostoje a1, a2, b1, b2, c1, c2 ∈ S takvi da vrijedi

m ≡ a1a2 + b1b2 + c1c2 (mod p).

Rje²enje. Prisjetimo se da je Zp zapravo polje, tj. svaki s ∈ Zp \ 0 imainverzni (tj. recipro£ni) element s−1. U kompaktnijoj skupovnoj notacijiºelimo dokazati

Zp = A · A+ A · A+ A · A.

Promotrimo funkciju f : Zp → C, f :=∑

s∈S χs·S, gdje χ ozna£ava karakte-risti£nu funkciju skupa. Naprije primijetimo da je

(f ∗ f ∗ f)(x) =∑

a,b,c∈Zpa+b+c=x

f(a)f(b)f(c) =∑

a1,b1,c1∈Sa,b,c∈Zpa+b+c=x

χa1·S(a)χb1·S(b)χc1·S(c)

=∑

a1,b1,c1∈Sa2,b2,c2∈S

a1a2+b1b2+c1c2=x

1 = broj traºenih prikaza od x

pa zapravo trebamo dokazati da je (f ∗ f ∗ f)(x) > 0 za svaki x ∈ Zp.Fourierova transformacija od f je

f(ξ) =∑s∈S

∑x∈Zp

χsS(x)e2πixξ/p = [x = sy] =∑s∈S

∑y∈S

e2πisyξ/p =∑s∈S

χS(sξ).

Imamo f(ξ) = |S|2. Za ξ 6= 0 su elementi sξ svi meusobno razli£iti kako svarira pa aritmeti£ko-kvadratna nejednakost i Plancherelov identitet daju

|f(ξ)| ≤ |S|1/2(∑s∈S

|χS(sξ)|2)1/2

≤ |S|1/2(∑ξ∈Zp

|χS(ξ)|2)1/2

= |S|1/2(p∑x∈Zp

|χS(x)|2)1/2

= |S|1/2p1/2|S|1/2 = p1/2|S|.

Zbog nejednakosti Minkowskog imamo(∑ξ∈Zp

|f(ξ)|2)1/2

≤∑s∈S

(∑ξ∈Zp

|χsS(ξ)|2)1/2

=∑s∈S

p1/2(∑x∈Zp

|χsS(x)|2)1/2

= |S|p1/2|S|1/2 = p1/2|S|3/2.

Kona£no, zbog (f ∗ f ∗ f )(ξ) = f(ξ)3 i formule inverzije imamo

(f ∗ f ∗ f)(x) = Re(f ∗ f ∗ f)(x) =1

pRe∑ξ∈Zp

f(ξ)3e−2πixξ/p

Page 66: Kako zapisati rje²enja? - Math · Kako zapisati rje²enja? IMO zadaci i bodovne sheme ... Nemojte zaboraviti trivijalne slu£ajeve (nulu, prazan skup, bazu indukcije) Pi²ite obja²njenja,

≥ 1

pf(0)3 − 1

p

∑ξ∈Zp\0

|f(ξ)|3

≥ p−1|S|6 − p−1p1/2|S|∑ξ∈Zp

|f(ξ)|2

≥ p−1|S|6 − p−1/2|S|p|S|3 = p−1|S|6 − p1/2|S|4

= p−1|S|4(|S|2 − p3/2

)> 0.

Naime, po pretpostavci zadatka je |S| > p3/4, tj. |S|2 > p3/2.

Princip neodreenosti

Za funkciju f : A→ C na kona£noj abelovoj grupi A ozna£imo

supp(f) := x ∈ A : f(x) 6= 0,supp(f) := ξ ∈ A : f(ξ) 6= 0.

Kratica supp dolazi od engleske rije£i support koja se na hrvatski prevodi kaonosa£ funkcije.

Teorem 14. (Princip neodreenosti; iz £lanka [1])

(a) Za svaku funkciju f : A→ C koja nije identi£ki jednaka konstanti 0 vrijedi

| supp(f)| | supp(f)| ≥ |A|.

(b) Za svaku funkciju f : Zp → C, gdje je p prost broj, koja nije identi£ki jednakakonstanti 0 vrijedi

| supp(f)|+ | supp(f)| ≥ p+ 1.

Obratno, za svake skupove A,B ⊆ Zp koji zadovoljavaju |A| + |B| ≥ p + 1

postoji funkcija f : Zp → C takva da je supp(f) = A i supp(f) = B.

Zadaci

3. Dokaºite (a) dio principa neodreenosti. (Dokaz (b) dijela je puno teºi inije sasvim elementaran.)

Rje²enje. Kori²tenjem nejednakosti trokuta, Cauchy-Schwarz nejednakostii Plancherelovog identiteta dobivamo

supξ∈A|f(ξ)| = sup

ξ∈A

∣∣∣∑x∈A

f(x)E(x, ξ)∣∣∣ ≤∑

x∈A

|f(x)| =∑

x∈supp(f)

|f(x)|

≤( ∑x∈supp(f)

12)1/2( ∑

x∈supp(f)

|f(x)|2)1/2

= | supp(f)|1/2(∑x∈A

|f(x)|2)1/2

Page 67: Kako zapisati rje²enja? - Math · Kako zapisati rje²enja? IMO zadaci i bodovne sheme ... Nemojte zaboraviti trivijalne slu£ajeve (nulu, prazan skup, bazu indukcije) Pi²ite obja²njenja,

= | supp(f)|1/2(|A|−1

∑ξ∈A

|f(ξ)|2)1/2

= |A|−1/2| supp(f)|1/2( ∑ξ∈supp(f)

|f(ξ)|2)1/2

≤ |A|−1/2| supp(f)|1/2| supp(f)|1/2 supξ∈A|f(ξ)|.

Ako f nije identi£ki jednaka 0, tada ni f nije identi£ki jednaka 0 pa jesupξ∈A |f(ξ)| > 0. Dijeljenjem s tim faktorom dobivamo traºenu nejedna-kost.

4. (Rezultat iz £lanka [1].) Neka je p prost broj i neka je S := z ∈ C : zp = 1skup p-tih korijena iz jedinice. Nadalje, neka su c0, c1, c2, . . . , cp−1 ∈ C,meu kojima je to£no k ≥ 1 brojeva razli£ito od 0. Dokaºite da je mogu¢eodabrati podskup T ⊆ S koji ima |T | = p − k + 1 elemenata i takav je daza svaki z ∈ T vrijedi

∑p−1j=0 cjz

j 6= 0.

Rje²enje. Promotrimo funkciju

f : Zp → C, f(x) :=

p−1∑j=0

cj(e−2πix/p)j.

Zapravo trebamo dokazati da postoji barem p − k + 1 razli£itih x ∈ Zptakvih da je f(x) 6= 0, tj. trebamo pokazati | supp(f)| ≥ p− k + 1.

Primijetimo da je

f(ξ) =∑x∈Zp

(∑j∈Zp

cje−2πixj/p

)e2πixξ/p =

∑j∈Zp

cj∑x∈Zp

e2πix(ξ−j)/p = p cξ.

Po pretpostavci zadatka je to£no k od brojeva f(0), f(1), . . . , f(p− 1) raz-li£ito od 0, tj. | supp(f)| = k. Po (b) dijelu principa neodreenosti imamo

| supp(f)| ≥ p+ 1− | supp(f)| = p− k + 1.

5. (Cauchy-Davenportov teorem) Ako je p prost broj i ako su A,B ⊆ Zp ne-prazni, dokaºite da tada vrijedi

|A+B| ≥ min|A|+ |B| − 1, p

.

Pritom, kao i obi£no, ozna£avamo

A+B := a+ b : a ∈ A, b ∈ B.

Rje²enje iz £lanka [1]. Najprije tvrdimo da moºemo na¢i skupoveX, Y ⊆ Zptakve da je

|X| = p+ 1− |A|, |Y | = p+ 1− |B|, |X ∩ Y | = max|X|+ |Y | − p, 1

.

Razlikujemo dva slu£aja.

Page 68: Kako zapisati rje²enja? - Math · Kako zapisati rje²enja? IMO zadaci i bodovne sheme ... Nemojte zaboraviti trivijalne slu£ajeve (nulu, prazan skup, bazu indukcije) Pi²ite obja²njenja,

(1) Ako je |A|+ |B| ≤ p+ 1, tada moºemo uzeti

X = 0, 1, . . . , p− |A|, Y = |B| − 1, |B|, . . . , p− 1.

(2) Ako je |A|+ |B| > p+ 1, tada moºemo uzeti

X = 0, 1, . . . , p−|A|, Y = p−|A|, p−|A|+ 1, . . . , 2p−|A|− |B|.

Prema (b) dijelu principa neodreenosti postoje funkcije f, g : Zp → C takveda je

supp(f) = A, supp(f) = X, supp(g) = B, supp(g) = Y.

Promotrimo funkciju f ∗ g. Iz denicije konvolucije

(f ∗ g)(x) :=∑y∈Zp

f(x− y)g(y)

se odmah vidi

supp(f ∗ g) ⊆ supp(f) + supp(g) = A+B,

dok iz svojstva(f ∗ g)(ξ) = f(ξ)g(ξ)

slijedisupp(f ∗ g) = supp(f) ∩ supp(g) = X ∩ Y.

Kona£no, kori²tenjem (b) dijela principa neodreenosti dobivamo

|A+B|+ |X ∩ Y | =∣∣ supp(f ∗ g)

∣∣+∣∣ supp(f ∗ g)

∣∣ ≥ p+ 1,

²to je upravo

|A+B| ≥ p+ 1−maxp+ 2− |A| − |B|, 1

= min

|A|+ |B| − 1, p

.

Zadaci za samostalan rad

1. Neka je n prirodan broj. Ozna£imo s Pn familiju svih 2n podskupova skupa1, 2, . . . , n. Za svaki S ∈ Pn dan je realni broj aS i pretpostavimo da jeto£no k ≥ 1 od tih brojeva aS razli£ito od 0. Dokaºite da jednadºba∑

S∈Pn

aS∏j∈S

xj = 0

ima najvi²e 2n(k−1)k

rje²enja u skupu −1, 1n, tj. ima najvi²e toliko n-torki (x1, x2, . . . , xn) koje zadovoljavaju jednadºbu i za svaki indeks j je ilixj = −1 ili xj = 1.Napomena: Za S = ∅ produkt

∏j∈S xj uvijek interpretiramo kao broj 1.

Naprimjer, za n = 2 jednadºba glasi

a∅ + a1x1 + a2x2 + a1,2x1x2 = 0.

Page 69: Kako zapisati rje²enja? - Math · Kako zapisati rje²enja? IMO zadaci i bodovne sheme ... Nemojte zaboraviti trivijalne slu£ajeve (nulu, prazan skup, bazu indukcije) Pi²ite obja²njenja,

Rje²enje. Promotrimo grupu

A = Zn2 = Z2 ⊕ · · · ⊕ Z2︸ ︷︷ ︸n

= 0, 1n

i uspostavimo bijektivnu korespondenciju

Pn → A, S 7→ karakteristi£na funkcija skupa S

s inverzom

A→ Pn, (z1, . . . , zn) 7→ j ∈ 1, 2, . . . , n : zj = 1.

Dakle, skupu S ∈ Pn odgovara n-torka z = (z1, . . . , zn) takva da je zj = 1za j ∈ S i zj = 0 za j ∈ Sc. Tada umjesto aS pi²emo az.

Deniramo funkciju f : A→ C,

f(y1, . . . , yn) :=∑S∈Pn

aS∏j∈S

(−1)yj =∑

z=(z1,...,zn)∈A

az

n∏j=1

(−1)yjzj ,

tj.f(y) :=

∑z∈A

az E(y, z).

Primijetimo da zapravo ºelimo pobrojati rje²enja (y1, . . . , yn) ∈ A jednadºbef(y1, . . . , yn) = 0. Trebamo dokazati da je | supp(f)| ≥ 2n

k, jer ¢e tada broj

rje²enja biti najvi²e 2n − 2n

k.

Zbog E(y, z) ∈ −1, 1 ⊂ R imamo

f(ξ) =∑y∈A

(∑z∈A

az E(y, z))E(y, ξ) =

∑z∈A

az∑y∈A

E(y, ξ − z) = 2n aξ.

Po pretpostavci zadatka je | supp(f)| = k. Zato (a) dio principa neodree-nosti primijenjen na grupu A daje

| supp(f)| ≥ |A|| supp(f)|

=2n

k.

Kori²tena literatura i dodatni izvori zadataka

[1] T. Tao, An uncertainty principle for cyclic groups of prime order, Math.Res. Lett. 12 (2005), 121127.

[2] T. Tao, V. Vu, Additive Combinatorics, Cambridge University Press, 2006.